Как стать автором
Обновить

Комментарии 339

Лично мне MWI до сих пор кажется костылём, который дюже нравится популяризаторам и авторам научной фантастики откуда и следует его чрезвычайная популярность. Мне по нраву больше интерпретация описанная в данной презентации:
www.youtube.com/watch?v=dEaecUuEqfc

Буду благодарен если кто-то профессионально разбирающийся в квантмехе пояснит нет ли там несоответствий/неточностей, или же противоречий с какими-либо исследованиями.

Отличная лекция. Но мне кажется, автор там почти не говорит про интерпретации, и всё что он говорит не противоречит MWI. (Но я её смотрел давно, и возможно что-то забыл).

Спасибо за статью! Интересно!..

Вопрос. Нет ли фразеологической неточности вот в этом предложении:
Фатальная проблема этого подхода в том, что декогеренция слабее, чем коллапс – она объясняет, почему мы не видим туманную смесь разных состояний котов, но не говорит, что остается только один кот!
?

не должна ли концовка быть такой:
… но не говорит, что остается кот только в одном состоянии!

или такой:
… но не говорит, что остается только одно состояние кота!

Ведь прямо перед этим говорится о смеси состояний котов, а не о наличии нескольких (физически? квантово? разных?) котов.
Спасибо
Исправил
Касательно опроса (прошу прощения за оффтоп). Местами набор вариантов слишком скудный. Допустим, в первом вопросе, если мне не изменяет память, есть вероятность, что мозг Мэри, не обученный различать цвета в период наибольшей пластичности, не сумеет это сделать, когда глаза их увидят.
Возможно, но проанализирую результаты в таком виде
Оу… Заметка для себя: надо обновлять ленту прежде чем постить статьи:)
Ну, больше ММИ — лучше.

Кстати:
И вот в феврале этого года костыль удалось убрать.

Так давным давно убрали, и не одним способом.

Ого, как мы синхронно!!!

Я, главное, уже месяца полтора думал, что надо бы написать, да все откладывал. А сегодня пришел на работу, а работа не идет, дай, думаю, наконец напишу разнообразия ради, пока не забыл совсем. А вот оно как сложилось:)
Квантовая запутанность налицо))

Уточню, что из трех статей только "Probabilities from Entanglement, Born's Rule from Envariance" 2005 года прошла пир-ревью.

В моих первая тоже прошла:
To appear (under the title «How to prove the Born rule») in Saunders, Barrett, Kent and Wallace, «Many Worlds? Everett, Quantum Theory, and Reality» (Oxford University Press)

Можно найти и книжку, только не в открытом доступе, поэтому ссылка идет на архив.

Ну книги не проходят пир ревью в том смысле, в котором его проходят статьи, хотя обычно и подразумевается, что информация там основана на достоверных источниках.

очень разумно создавать целую вселенную только для того, что бы дать вам шанс ограбить банк )))
Каждую секунду создавать новую вселенную только если вы вдруг решите ограбить банк. )
Это главный минус данной интерпретации
НЛО прилетело и опубликовало эту надпись здесь
Только чуть медленнее — смотря на суперпозицию котов, вам надо дождаться фотонов от них
НЛО прилетело и опубликовало эту надпись здесь

Когда речь заходит о создании Вселенной, критерии разумности могут отличаться от привычных.

Скажем, энтропия такой квантовой вселенной (казалось бы, сумма бесконечностей) получается равна... нулю.

Спасибо за статью!
А вас не затруднит так же популярно объяснить, чем плоха теория де Бройля-Бома (она же pilot wave theory)? Как-то я стал натыкаться на ее упоминания довольно часто, но поскольку сам в квантмехе не смыслю, понять, что же с ней «не так» — не могу.

Я не специалист, но по-моему она не даёт хорошего объяснения запутанности частиц. Грубо говоря, в ней невозможен квантовый компьютер.

Да в целом все «так», это хорошо разработанная теория. eterevsky Запутанность она отлично объясняет.
Проблемы в ней в основном со связями со специальной и общей теорией относительности: так как теория принципиально нелокальна и позволяет взаимодействие между скрытыми параметрами со скоростью выше скорости света, ее сложно сделать совместимой с ограничениями СТО.
Один пропонент ее хорошо сравнил с MWI — MWI это дерево, в теория Бома это дерево, по которому ползет муравей. Ветка, по которой он ползет 'настоящая' а остальные — нет. С философской точки зрения это непонятно — глобальная волновая функция такая же, в пустых ветвях люди встречаются, старадают, любят друг друга, но это как то «понарошку»

Вообще, из того, что я прочитал в этой статье, я не вижу процесса создания второй вселенной. Да, создаются 2 кота, и только один из них остается в нашей вселенной. Что происходит со вторым — пока неизвестно, может, он, в некоторой степени "мнимый", может, он отправляется во времени назад (!), а может его съедают лангольеры )

Почитайте про декогеренцию. Все таки формат статьи Хабра слишком короткий, чтобы изложить это
Спасибо!
детерминированной, объективной и локальной
детерминированной = предопределённость?
Означает ли это что мир/вселенная предопределена и поскольку (ну или если) мысль базируется на физическом носителе то следовательно нет свободы воли?
Да есть вроде как эксперименты что сознание это лишь следствие работы мозга, мозг сам принимает решения, то есть сознание не является причиной решений. Теперь получается что если бы был супер-супер компьютер, он мог бы вычислить заранее любые действия любого человека или групп людей или животных, то есть всё предопределено, можно расслабить булки и ни к чему не стремится? :-)

Да и кстати, как случайность живет с детерминированностью, одно другое ведь на 100% исключает?
«если бы был супер-супер компьютер, он мог бы вычислить заранее любые действия любого человека или групп людей или животных, то есть всё предопределено»

— Сразу возникает вопрос — а есть ли такой компьютер? Вообще — возможен ли он? И, если возможен — возможно ли его взломать?
«если бы был супер-супер компьютер,.." "— Сразу возникает вопрос — а есть ли такой компьютер?"
Вселенная — она и есть такой суперкомпьютер!?
Я к тому что слово «детерминированной» означает что если загнать все параметры всех частиц и полей в компьютер, то можно вычислить что человек сделает через год, гадалки смогут предсказывать 100% наперед всё что произойдет, если исключить эффект бабочки, который тоже предопределен (включен в план будущего) этой вселенной, а значит заведомо эффект бабочки исключен, так как это не побочный эффект, а план/программа вселенной.
"слово «детерминированной» означает что если загнать все параметры всех частиц и полей в компьютер, то можно вычислить что человек сделает через год, гадалки смогут предсказывать 100% наперед всё что произойдет"

— Для этого компьютер гадалки должен быть размером со Вселенную… Ведь надо вычислить вообще всё, от каждого атома и фотона звёзд, до гипотетических планкеонов заполняющих вакуум. И это мы ещё не знаем — есть ли что-то снаружи нашей Метагалактики! И не взаимодействует ли она с другими пространствами.

Для вычисления состояний вселенной нужна другая, соизмеримая по размерам вселенная. Так что теоретически такой компьютер невозможен.

"гадалки смогут предсказывать 100% наперед всё что произойдет, если исключить эффект бабочки"

— Если бы такая гадалка с суперкомпьютером существовали бы, то эффекта бабочки для них не существовало бы — потому как это было бы такое же детерминированное действие, только добавляющее переходы. При обнаружении путешественника компьютер достаточно пересчитать всё с нужного момента и ситуация снова станет предсказуемой.
Еще этот компьютер должен вычислять процесс вычисления процесса вычисления и т.д.
Рекурсия
Тут ещё есть тонкость, и без рекурсий:

1 — любое вычисление требует энергии, и чем выше уровень детализации физики, тем выше будет потребление вычислительных мощностей и энергии. Вон как биохимики маются с вычислением «простейших» атомарных систем моделируя биологический вирус. Даже обсчитывая белки и то приходится идти на хитрости с редукционизмом, чтобы обсчитывать несколько тысяч (десятков, сотен тысяч) атомов.

2 — гипотетический компьютер супергадалки, обсчитывающий ВСЕ взаимодействия Метагалактики, определённо должен нарушать и хорошо известные законы физики. В частности причинно-следственный, который есть ограничение на скорость вычислений (планка скорости передачи информации).
Возможен и не возможен одновременно.

То есть, если есть метавселенные, которые влияют на нашу вселенную, но мы не можем влиять на них (в принципе!), то это не очень хорошая концепция, так как тут что-то не то с сохранением энергии информации (точно не скажу, но интуитивно как-то так). Получается, мы влияем на эти метавселенные, а тогда это уже формально одна вселенная, всё просто. Суть видимой вселенной как раз в том, что это ровно то, до чего мы можем дотянутся, остальное может существует, может нет — в целом без разницы.

Что до свободы воли — её нет, смиритесь. Просто расслабьтесь и получайте удовольствие ;) Хотя если быть точнее, тут возможны вариант, навроде того же компатибилизма и его вариаций.

В принципе, нет околоникаких сомнений в том, что наша вселенная — это компьютер. Тот или иной. За это довольно много сильных косвенных доводов, плюс рядом приютилась теория вероятностей. Получается довольно много неприятных нефальсифицируемых гипотез, но в тоже время все они сводятся именно к идеи вселенной-компьютера, причём с абсолютным пофигизмом на любые метавселенные до тех пор, пока мы друг на друга не влияем.

Таким образом да. Имея ВСЕ параметры (что технически вряд ли возможно, так как само пространство-время есть параметр), можно запустить компьютер, который вычислит полное дерево событий до и дерево событий после (и рекурсивно все возможные события сейчас, хотя их число растёт экспоненциально, так что возможно их рекурсивно-бесконечно). Однако. Время вычислений без влияния мета-вселенных будет большим, чем время, до которого нам требуется досчитать, а с влиянием мета-вселенных нам придётся считать и их, что возвращает нас к невозможности посчитать некоторое событие ДО того, как оно наступит…

Короче, компьютер возможен (он уже в принципе построен, но масштабы пока недостаточно велики для астрономических). И даже более того, мы почти наверняка его построим, что в общем-то дорога в рекурсивные вселенные, а это дичайший кризис для философии. А вот его полезность — весьма сомнительна…

В любом случае всё вышенаписанное диванный бред, так что. Будем посмотреть)
НЛО прилетело и опубликовало эту надпись здесь
Нет не исключает.
Глобально она детерминирована (birds view, для бога так сказать)
Локально для внутренного наблюдателя нет

Приведу пример. Для тестера игры квеста, который обязан пройти ВСЕ ветки квеста и проверить ВСЕ возможности, игра детерминирована. А для игрока — нет.
НЛО прилетело и опубликовало эту надпись здесь
ну как бы зависящее от времени уравнение Шредингера — дифференциальное уравнение
НЛО прилетело и опубликовало эту надпись здесь
Что такое измерение? Какие то взаимодействия между квантовыми объектами магическим образом не описываются квантовой механикой, так как на них заклятье «Йа измерительный прибор»? Снова призраки копенгагена
НЛО прилетело и опубликовало эту надпись здесь
НЛО прилетело и опубликовало эту надпись здесь
НЛО прилетело и опубликовало эту надпись здесь
Как раз с математическим аппаратом там нет никаких проблем
Все вычисляется с громадной точностью и соответвует результатам эксперимента
НЛО прилетело и опубликовало эту надпись здесь
Нет никакой войны. Все эксперименты пока что подтверждают копенгагенскую интерпретацию.
Список — в студию!

Интерпретация ведь не делает никаких предсказаний, потому что она, сюрприз, интерпретация. В них можно верить или не верить, это дело каждого. С точки зрения физики разницы никакой.

Это не вопрос веры. Физика экспериментальная наука. Результаты экспериментов совпадают с предсказаниями копенгагенской интерпретации. Собственно само понятие каких либо интерпретаций лишено смысла. Это типа как по другому интерпретировать законы Ньютона.

Результаты экспериментов совпадают с предсказаниями любой интерпретации.

и какие эксперименты подтверждают теорию скрытых параметров, которой придерживался Эйнштейн или ту же многомировую интерпретацию?

Я же написал — все. Разные интерпретации не предлагают разные предсказания или разные математические модели. Они только интерпретируют их.

теория которая не предлагает предскааний, то есть не представляет математический апарат который может просчитать результаты и подтвердить экспериментом, это не научная теория.
В противном случае все интерпретируется очень просто — на все воля божья.
И математические модели как раз очень даже разные. Даже в рамках класической квантовой механики предсталение Шредингера и представление Гейзенберга это разные математические модели хоть и пересчитываемые друг в друга.
Или что по вашему интерпретация? Обяснение преподавателя с удачными аналогиями и метаформаи?

Можете ли вы предложить эксперимент который фальсифицирует копенгагенскую (или многомировую) интерпретацию, но не фальсифицирует квантовую механику как таковую?

НЛО прилетело и опубликовало эту надпись здесь
Я же написал — все.
Эксперимент не может подтвердить или опровергнуть интерпретацию, только теорию.
Да. Поэтому интерпретации не имеют смысла. Либо они повторяют то же самое но другими словами то есть это та же теория, либо это другие теории.
Однако смотрите. Представьте такую интерпретацию: все соответсвует квантовой механике, но за волновой функцией следят разноцветные летающие единороги с калькуляторами, они производят вычисления и заставляют волновую функию подчиняться уравнениям

Считаете ли вы что подобная интерпертация имеет те же права, что и MWI или Копенгаген?
Я не говорил что вообще нельзя принимать или отвергать интерпретации. Эксперимент этого сделать не может, а размышления могут.
Разумеется, имеет. Более того, данный вопрос не имеет к собственно квантовой механике никакого отношения. Что касается самой статьи, у меня осталось впечатление, что всё это — развитие известного утверждения Мермина («Shut up and calculate»).
Я вот концовку не совсем понял: как MWI противоречит спутанности (entanglement) и корпускулярно-волновому дуализму, или объясняет их? Как MWI объясняет неравенство Белла?
Не, я конечно сам приверженец миров Эверетта, но не понимаю в чем проблема с другими интерпретациями. Это всего лишь интерпретации. Мат аппарат один и тот же. Миры Эверетта не нужно понимать как на какие-то параллельные независимые измерения. Это состояние суперпозиции, разложенное по базисным состояниям. Да, на эти базисные состояния можно (интуитивно) смотреть как на отдельные миры, но нужно то понимать, что мир всего один, и он квантовый…
Неравенство Бэлла имеет hidden assumption, одну историю. В MWI Alice и Bob имеют разные версии. MWI не имеет проблем с entanglement (она локальна) и в ней нет корпускулярно-волновому дуализма так как все есть волны, а частицы — иллюзия, порожденная decoherence
Ага, с запутанностью понятно. Но с MWI тоже есть одна проблема, по крайней мере в том её варианте, который описан в википедии: " невзаимодействующих между собой параллельных вселенных". Проблема в том, что это не позволяет объяснить явление интерференции электрона самим с собой при прохождении через две щели.
ну да ведь траектории в разных мирах
Тут как раз все тривиально. Интерференция происходит в одной «ветке», а декогеренция как раз интерференцию и разрушает (это главное страдание создателей квантовых компьютеров.
Если всё происходит в одной ветке, то какой тогда смысл в других ветках (и соответственно в MWI)? Или в копенгагенской интерпретации нет декогеренции? В общем пока не совсем понятно чем ваш вариант MWI отличается собственно от копенгагенской.
Ну да, в первых версиях «копенгагена» декогеренции не было, но потом всё ж добавили. И вообще декогеренция не привязана к какой-либо интерпретации. По крайней мере не вижу, чтобы в MWI она объяснялась как-то проще, чем не в MWI.

Так все происходит во всех ветках

>Ну да, в первых версиях «копенгагена» декогеренции не было, но потом всё ж добавили.

Я как раз писал про фатальную проблему с этим подходом
НЛО прилетело и опубликовало эту надпись здесь
и аж целое разделение вселенной (т.е. как бы копирование)


Резделение идет постепенно и это физический процесс
В сторогом соответствии с формулами QM — в отличие от коллапса, который есть полная придумка человека
Я бы еще сказал что выносящий мозг <> абсурдный
MWI да, выносит мозг. Но логична
Как раз они и взаимодействуют через интерференцию.
Но с MWI тоже есть одна проблема, по крайней мере в том её варианте, который описан в википедии: " невзаимодействующих между собой параллельных вселенных". Проблема в том, что это не позволяет объяснить явление интерференции электрона самим с собой при прохождении через две щели.

Просто параллельных вселенных никаких нет, и веток никаких нет (это все некорректные аналогии для упрощения) — есть одна ВФ, которую можно, проецируя, разложить на компоненты, а потом назвать такие компоненты "мирами". Естественно, ни в каком сколько-нибудь строгом смысле они мирами на самом деле не являются, а разложение не имеет никакого физического смысла — ВФ как была одна, цельная, так цельной и остается.

Ну это то понятно. Там вопрос в другом был — чем это отличается от «копенгагена».

В "копенгагене" есть два способа, которым может меняться состояние системы — это, во-первых, стандартная динамика по у-ю Шредингера, а во-вторых — коллапс волновой ф-и, который происходит при взаимодействии.
В мми есть только динамика по Шредингеру и все, никакими другими способами волновая ф-я измениться не может.
При этом формулы и предсказания, конечно, одинаковые в обоих случаях, иначе бы это были разные теории, а не разные интерпретации одной теории.

ок, спасибо. Не сразу уловил суть. Пришлось глянуть статью, оригинальную в смысле. Да, там авторы выводят постулат об измерениях. Доказательство конечно сложное, надо разбираться. Но если оно верное, то да, это определенный аргумент в пользу MWI.

В соответствии с многомировой интерпретацией сознание как игла по пластинке скользит по локально недетерминированной дорожке. Но если подумать о сознании, одновременно проживающем все варианты, при этом имеющем множество личностей (хранилищ паияти)… тут и начинается фантастика :)

НЛО прилетело и опубликовало эту надпись здесь

Да, это сознание дробится. И это реально выносит мозг...

Почему-то люди любящие пофилосовствовать на тему интерпретации КМ не в курсе корректного ответа на вопрос «как так?». А он таки есть — и уже давно.
Алгоритм:
1) читаем книжку нобелевского лауреата Пригожина под названием «время, хаос, квант — к решению парадокса времени»
2) понимаем, согласно изложенному в книжке и вполне строго доказанному, что чем быстрее и чаще происходят квантовые взаимодействия в сложной системе (состоящей из большого кол-ва квантовых частиц), тем уже возможный спектр состояний системы.
3) сюда добавляем понимание что такая штука как гравитация вообще ничем и никак не экранируется, и остаётся таки взаимодействием происходящим очень часто — значит нет никакой многомировой вселенной, равно как и нет никакого коллапса. Есть система в которой есть почти бесконечное множество волновых функций (описывающих вселенную) с почти бесконечно-близкими макроскопическими состояниями. Так что с точки зрения макро-состояния, вселенная она она. А с точки зрения микросостояния — это дико большое кол-во дико похожих друг на друга волновых функций. По сути вопрос математического формализма, не более того — считать вселенную классической и единственной, или считать её физически-реализованной дельта-функцией образованной из всего спектра её состояний. Физикам проще первое, математикам возможно что и второе. А отклонение к виде «в одной вселенной кот жив а в другой мёртв» математически стремится к нулю, и стремится крайне быстро — т.е. в практическом смысле так сказать смысла не имеет.
4) Profit!
Рассмотрим следующую ситуацию. Берем сферического Ш-кота в вакууме и помещаем его в свободное пространство, нет ничего вовне этой конструкции. Кот, соответственно, не жив и не мертв, то есть оба состояния равновероятны. Пусть рядом сформируется источник некогерентных фотонов — лампа. Разрушит ли это волновую функцию кота?
Если да, то почему? Если нет, то далее, кот отражает фотоны и они разлетаются по вселенной неся волновую функцию свою и кота. Спутаны ли фотоны?
Если да, то почему? Если нет, то предположим, впоследствии, где то там в пространстве, зарождаются наблюдатели. Происходит контакт фотона с наблюдателем и коллапс волновых функций, всех двух? Или одной?
Не получается ли в итоге, что два независимых в простанстве и времени фотона, связаны некой, третьей волновой функцией? И каждый фотон, несет несметное количество волновых функций. Вселенная меняется, в момент когда мы смотрим на звезды?

P.s. Еще более интересно, если от состояния системы Ш-кота, зависит, прошел/отразился ли фотон в направлении наблюдателя. Коллапсирует ли волновая функция вообще без контакта с фотоном? Значит ли это, что волновая функция фотона распространяется по всем вероятностным путям. То есть, все влияет на все?

Во первых фотоны не отражаются а переизлучаются. Это будут лругие ыотоны. И таки да если состояние фотона будет зависеть от состояния кота (то есть будет проведено измерение) то фотон и кот будут в запутанном состоянии и находится в состоянии суперпозиции с точки зрения стороннего наблюдателя
Еще чуть упростим. Нет кота, есть некий экран который падает на лампу и закрывает путь фотонам. Соответственно, закрывает и не закрывает одновременно. Пока экран не упал, нет взаимодействия с фотонами, и нет измерения. Или есть?
Измерять можно и без физического взаимодействия (surprise, surprise!)
Погуглите Quantum Bomb tester
тем не менее это взаимодействие поскульку имеет место получение информации. Датчик бомбы влияет на вероятность прохождения фотона хоть и не на сам фотон.
Одна из фишек КМ — достаточно присутствия наблюдателя и потенциальной возможности измерения чтобы повлиять на исход события.
В даном случае неисправная бомба не является наблюдателем поскольку не влияет на вероятность исхода событий а исправная является даже если фотон не пошел чрез датчик потому что в таком случае иы знаем что он пошел по другой стороне
Погуглил, проникся. Мозг сломан, отказывается создавать под такое модель мира.
Неисправная бомба по условиям эксперимента не обладает датчиком фотона. Исправная обладает и поглощает его. Разница в поведении вовсе не в возможности взорваться, а в наличии или отсутствии датчика. Классический вариант попытки “поймать” электрон в случае двухщелевого эксперимента.
А что мешает неисправной бомбе иметь датчик. На самом деле такие технические детали не имеют значения. Бомба или реагирует на фотон или нет потому что неисправна.
Вот здесь наглядно описывается эксперимент
Мешает то, что датчик будет выступать в роли “Наблюдателя” и весь смысл теряется. Нерабочая бомба не имеет датчика по определению эксперимента — иначе не возникла бы интерференция.
не знаю что тут упростилось — например непонятно теперь что измеряется.
Спор физиков о том, какая интерпретация квантовой механики верна, это как спор представителей монотеистический религий о том, чей Бог главнее.
в отличие от интерпретаций наличие какого либо бога или всех вместе взятых нельзя подтвердить эксперимнтом или хотя бы данными наблюдений.
а как подтвердить интерпретацию экспериментом, вот тоже интересный вопрос)
ну например нарущение неравенств белла подтверждено экспериментально. Ну или класика — двухщелевой эксперимент и интерференция частицы сама с собой
Это проверка квантовой теории. Но как эти эксперименты повлияли на то, какую интерпретацию выбрать?
Просто не надо путать саму квантовую механику с её интерпретациями, то есть с попытками объяснить решения уравнений на понятном человеку языке.
Например «Квантовое самоубийство» Тегмарка вполне себе эксперимент разделяющий результаты для копенгагенской и многомировой интерпретаций.

Интерпретация все-таки предполагает n фактов об устройстве мироздания которые бы связывали уже выясненные данные и законы, иногда(как в этой ситуации) эти факты все-таки проверяемы.
Не могли бы вы описать «эксперимент разделяющий результаты для копенгагенской и многомировой интерпретаций»?..
Ибо то, что обычно называют «квантовым самоубийством», стало бы доказательством многомировой интерпретации только в случае получения на эксперименте бесконечной продолжительность жизни его участника, но бесконечности в принципе не могут являться результатами реальных экспериментов, а значит доказать он ничего не может.
Совсем не так. У того же М. Тегмарка показано, какие в эксперименте с «квантовым бессмертием» самые невероятные вещи будут происходить уже совсем и совсем скоро, которых вполне достаточно для её подтверждения с высокой долей вероятности. Проблема там в другом — «неудачный эксперимент» не опровергает её, а постулирует, что те кто его наблюдает просто оказались не в том ответвлении с эксперементатором, а «удачный» — всегда ассоциирован с только участником эксперимента по определению, и, по сути, надёжно «убеждает» только его.

Эксперимент можно попробовать обобщить до «коллективного квантового бессмертия» — т.е. чтоб от квантового события срабатывало не одно ружьё для одного эксперементатора, а, например, «сверхмощная термоядерная бомба», т.е. чтоб при её срабатывании исчезали все локально связанные наблюдатели. Тогда факт их длительного существования будет столь же надёжным свидетельством многомировой интерпритации. Отсюда следует, что если найти в природе (желательно в нашей окрестности) такой объект или систему объектов, который в зависимости от квантового события, вызывает надёжное уничтожение в локальной области всех потенциальных наблюдателей(но почему-то до сих пор этого не сделал, так же как ружье всё-время даёт почему-то даёт «осечку») — теорию тоже можно считать с высокой вероятностью доказанной. В общем, надо просто поискать «ружьё»)

Учитывая, что человечество в принципе существует в том мире, который мы знаем — возможно, что это единственное логически непротиворечивое объяснение почему именно.
И это может быть решением парадокса Ферми
Парадокс Ферми неплохо разрешается и тем простым фактом, что космос огромен, а скорость света конечна и не так уж и высока. Человечество радио изобрело не так давно, и волны, которые мы излучили не так уж и далеко разошлись. Учитывая принцип заурядности, наши коллеги по разуму ничем лучше нас, вот и напоминаем мы пока что аборигенов, которые одни в Австралиии, а другие в Северной Америке лупят палкой по берегу окена пуская «сигналы» и пытаются выловить «ответные волны» от такой же палки обратно во время шторма. И это только по разрозненность в пространстве. А если взять во времени. Динозавры жили кратно дольше чем всё человечество. Условно говоря, мы не сталкиваемся с другими цивилизациями по той же причине, по которой «звёзды» в разных галактиках(да и в одной и той же) тоже не особо часто «сталкиваются». Места разместиться во Вселенной всем хватает.
НЛО прилетело и опубликовало эту надпись здесь
А признаков того, что хоть одна цивилизация в нашей галактике прожила достаточно долго для того, чтобы заполонить её — нет ни одного.


Давайте будем строги — надёжно «зарегистрированных» признаков нет ни одного. Но так и гравитационные волны были всегда — а зерегистрировали их недавно. Экзопланеты по одной считать вот вот только начали… И.т.п и т.д.

Да и вообще, весь «парадокс Ферми» по своей сути сформулирован глупо и нелепо. Больше похож на шутку или анекдот, чтоб его всерьез обсуждать. Только что звучит солидно. Какой-то «суслик из ДМБ» — только наоборот. www.youtube.com/watch?v=EHX7NZS8zAI
НЛО прилетело и опубликовало эту надпись здесь
Если честно — я не знаю. То, что пока нет признаков — это факт. То, что они могут позже появиться — причин этому не случиться пока нет. Какие точно значения вероятностей — я даже затрудняюсь понять как корректно их считать, а не то что бы оценивать конкретные значения. Может ли всё это продолжаться до бесконечности? — Скорей всего нет, т.к. мы как вид конечны по времени существования. Можем ли мы за время существования нашего вида никогда об этом не узнать? Запросто. Динозавры не дождались нашего появления. Нет никаких причин, указывающих, что мы должны получить точный ответ на этот вопрос(как и на любой другой). Что так же не исключает того, что этот ответ в принципе получить тоже возможно или невозможно.
НЛО прилетело и опубликовало эту надпись здесь
Варианты вполне себе есть. Самое очевидное — нет никакой «другой физики», сверхсвета, телепорта, нанороботов и зондов фон Неймана. А если где то там, на Альфацентавре, выпустили уже 128 модель айфона и живут по 300 лет, узнать мы об этом никогда не сможем.

Вариант два — Солнце, типичная звезда третьего поколения. По многим данным, первые два не дают обитаемых в нашем понимании планет. Быть может, в своем световом конусе, мы первая цивилизация космического уровня. Поле зеленеет боле-менее равномерно, быть может и вселенная то же. Гипотеза подтвердиться, если в Солнечной системе будет найдена простейшая жизнь (либо следы былой жизни) альтернативного происхождения. Скорее всего, в ближайшие 100-500 лет мы это проверим.
НЛО прилетело и опубликовало эту надпись здесь
Таки пока нет. В настоящее время, любой организм есть часть единого организма «биосфера Земли». Мы (в том числе человек, как часть биосферы) не можем послать клетку на другую планету и создать там биосферу. Можем только чуть чуть там прожить, подкармливая из пуповины.

Да, мы двигали отдельные атомы и написали ими слово из трух букв. Затраты на это чудовищные, в том числе энергетически. Собрать из атомов что то осмысленное, мы скорее всего не сможем, а если когда нибудь и сможем, это превысит все временные и энергетические рамки. Аналогично и синтез атомов, теоретически возможно, энергетически проще полететь и собрать то что звезды раскидали.

Световой конус работает в обе стороны! Не только нас не видят за 100 световых лет, но и мы. И чем дальше мы смотрим, тем раньше должна была зародиться цивилизация, и тем ниже соответственно шансы. Грубо говоря, на Альфацентавре мы увидим свой технический уровень, на другом краю галлактики — каменный век.
НЛО прилетело и опубликовало эту надпись здесь
Пишут, что «Вечные аквариумы» ограниченны сроком жизни креветок. Научные эксперименты «Биосфера 1-2-3» потерпели неудачу.

Бактерии собирают белки из аминокислот таки, не из атомов. Синтез в оружии всего одного типа и не особо контролируемый. До синтеза произвольного элемента таблицы менделеева, к примеру, из водорода, человечеству пока невообразимо далеко. То есть даже нет идей, как именно это может происходить в «домашних условиях».

Если наша цивилизация в майнстриме (а это наиболее вероятно) то с цвеличением отклонения его частота резко падает.
НЛО прилетело и опубликовало эту надпись здесь
самые невероятные вещи будут происходить уже совсем и совсем скоро, которых вполне достаточно для её подтверждения с высокой долей вероятности.

Будьте добры описать конкретнее…

Тогда факт их длительного существования будет столь же надёжным свидетельством многомировой интерпритации.

Даже близко не будет!
Это докажет лишь то, что вам очень повезло.
Вероятность выиграть в лотерею 6 из 45 примерно такая же, как вероятность, что конкретный радиоактивный атом проживёт в 16 раз больше, чем его время жизни. Как мы знаем, в лотерею выигрывают регулярно, так что нет ничего такого особенного в том, что вы проживёте в таком опыте в десяток раз больше «времени жизни».
И даже если вы проживёте в двадцать раз больше «времени жизни» — всё равно такое вполне объясняется просто случайностью. В конце концов вероятность того, что при зачатии появитесь именно вы, а не кто-то с другими генами, была ещё радикально ниже, но ведь в итоге появились именно вы!
Будьте добры описать конкретнее…


Я сейчас на память все не помню, но в книке Тегмарка «Математическая Вселенная» они подробно изложены. Думаю, что не очень сложно найти его описание эксперимента. Вроде того, что через 5 минут вы увидете падающий метеорит и т.п. Эти штуки логически появляются из-за получаемых вероятностей событий.

Даже близко не будет!
Это докажет лишь то, что вам очень повезло.


Вы вроде бы начали опровергать, а в конце взяли и всё сами подтердили:) Дело в том, что не нужно путать причину и следствие там, где этой связи нет. При кроссинговере получается, тот кто получается, и никакой задачи сделать там «тебя» или «меня», поэтому рассуждение в русле «при зачатии появитесь именно вы, а не кто-то с другими генами, была ещё радикально ниже» — это лишённая смысла последовательность букв.

Одна из существенных проблем эксперимента, это «участник никогда не сможет рассказать об этих результатах, так как с точки зрения стороннего наблюдателя, вероятность исхода эксперимента будет одинаковой и в многомировой, и в копенгагенской интерпретациях.»

Но вот какой нюанс — проблема возникает, из-за того, что участник и сторонние наблюдатели как-бы оказываются в разных «ответвлениях». Но вспомним «парадокс Вигнера» ( ru.wikipedia.org/wiki/Кот_Шрёдингера ), там квантовую систему образуют не только сам «кот в коробке», но так же «код в коробке и друг, который знает его состояние после её открытия». Очевидно, что рекурсивно можно обобщить на всех существующих наблюдателей. т.е. всех ноблюдателей можно запросто поместить в одну упряжку. Ещё один нюанс — дискретность наблюдателя, это условность, и чисто биологически мы скорее симбионты даже на уровне отдельного биологического организма, а уж коллективно и подавно.

Исходя из вышесказанного, понятие участника эксперимента можно обобщить и провести эксперимент «коллективного бессмертия» — где «участником» будут все локально связанные наблюдатели, а в качестве «ружья» будет выступать что-то помощнее, например «бомба» — главное, что при наступлении квантового события гарантированно перестают существовать все эти локально связанные наблюдатели. Как видим, суть эксперимента нисколько не изменилась.

Тогда проблема элегантно решается — факт длительного существования (как следствие получаем бессмертие) любых наблюдателей в присутствии «бомбы» явяляется серьёзным свидетельством в пользу ММИ. Как «загадочно» постоянно будет давать «осечку» ружьё, так и «загадочно» не будет срабатывать «бомба». И не надо кому-то одному рисковать своей головой.

С практической точки зрения всё что нужно, это найти и описать в природе в нашей локальной окрестности такой длительно существующий физический объект, который зависит от квантового события и приводит к процессу с взрывообразным эффектом нужной разрушительности (по аналогии, как квантовый эффект убивает кота в коробке).
Дело в том, что не нужно путать причину и следствие там, где этой связи нет. При кроссинговере получается, тот кто получается

В вашем эксперименте тоже самое: получается то, что получается.
Если получается, что вы ещё живы — ну вот просто так получилось и всё, никакой связи ни с чем тут нет.

Тогда проблема элегантно решается — факт длительного существования (как следствие получаем бессмертие) любых наблюдателей в присутствии «бомбы» явяляется серьёзным свидетельством в пользу ММИ. Как «загадочно» постоянно будет давать «осечку» ружьё, так и «загадочно» не будет срабатывать «бомба».

Блин, вы вообще читали моё прошлое сообщение?.. Если нет, повторю ещё раз:
Вероятность выиграть в лотерею 6 из 45 примерно такая же, как вероятность, что конкретный радиоактивный атом проживёт в 16 раз больше, чем его время жизни. Как мы знаем, в лотерею выигрывают регулярно, так что нет ничего такого особенного в том, что вы проживёте в таком опыте в десяток раз больше «времени жизни».
И даже если вы проживёте в двадцать раз больше «времени жизни» — всё равно такое вполне объясняется просто случайностью.
Блин, вы ...


Зря вы нервничаете. Похоже, что у нас качественно разное понимание сути эксперимента. Поэтому, все ваши довода просто лишены смысла в контексте моих рассуждений — незачем их цитировать. Предположу, что так же вы вряд ли хотите разбираться в сути как самого вопроса, так и в сути наших разногласий его интерпритации, поэтому не думаю, что есть смысл продолжать о чём либо спорить.
Тогда проблема элегантно решается — факт длительного существования (как следствие получаем бессмертие) любых наблюдателей в присутствии «бомбы» явяляется серьёзным свидетельством в пользу ММИ.

Всего-то осталось — сделать такую бомбу, а потом на ней подорваться :)

«Со мной-то все будет в порядке. А вот моя жена Анжелика останется вдовой.» — Макс Тегмарк

Кстати, вроде в этой вселенной он разводился...

Со мной-то все будет в порядке.

А с чего он взял что с ним? Может как раз не с ним, а с тем товарищем, у которого жена Анжелика :)


На самом деле ММИ не "разрешает" риски убийства а наоборот запрещает их. Т.к. любой маломальский риск обозначает, по факту, убийство (хоть и в другом мире). А случае если рискуете вы — вы даже выиграв в русскую рулетку по факту совершаете массовое самоубийство, так как несчетное множество вас "из других миров" проигрывает.

А с чего он взял что с ним?


По определению всегда найдётся тот, в котором с ним будет всё в порядке. Остальные его лично не интересуют. И миры между собой не сорвенуются, поэтому никто не проигрывает. Они несравнимы и говорить о чьей либо субъектности в паралельных мирах некорректно.
По определению всегда найдётся тот, в котором с ним будет всё в порядке.

А какая мне разница, если им буду не я?

А какая МНЕ разница, если им буду не Я?


Такой опции не предусмотрено. Это всё ВЫ. Вас же не смущает, что секунду назад или вперёд это уже не вы от вас сейчас. Или сделав шаг в сторону это тоже не вы. Всё вы.
Противоречие возникает только из интуитивного желания мозга воспринимать вас в ответлених как иную субъектность, а это не так. Обычное когнитивное искажение.
Такой опции не предусмотрено. Это всё ВЫ.

А умер кто тогда?


Противоречие возникает только из интуитивного желания мозга воспринимать вас в ответлених как иную субъектность, а это не так.

Если у нас разная история (один умер, а другой жив), то субъектность вполне определенно разная.

А умер кто тогда?

А как вот один и тот же фотон в опыте Юнга одновременно в двух местах бывает? Если сейчас он «тут», то кто «там»?

то субъектность вполне определенно разная.

А я вот не уверен в этом. Тут надо подумать как определить понятие «субъектности» в контексте MMI. Тогда можно будет подумать над тем разная одна или одна и та же. Логично предположить, что раз квантовое событие вызывает «расщепление миров», то вы получаете не «две разных истории»(один умер, а другой жив), а «суперпозицию двух разных и историй» и, соответственно и на выходе получите не другую «субъектность», а какую-то её суперпозицию(что бы это не значило). В общем, не думаю, что с классическим дискретным подходом(либо то, либо другое) смотреть на ситуацию со стороны логически-непротиворечиво будет так просто.
А как вот один и тот же фотон в опыте Юнга одновременно в двух местах бывает? Если сейчас он «тут», то кто «там»?

Один и тот же фотон и там и там. С-но, если я умер в каком-то мире — то я умер.
Вы почему-то используете эту логику только для определения того, кто остался жив. Но тут же полная симметрия.


а «суперпозицию двух разных и историй» и, соответственно и на выходе получите не другую «субъектность», а какую-то её суперпозицию(что бы это не значило)

Ну отлично. В половине этой суперпозиции я мертв, а в половине — жив. Почему вы учитываете ту половину, где жив, но отбрасываете ту, что мертв? Это как стакан, который на половину пуст и на половину полон.

НЛО прилетело и опубликовало эту надпись здесь
Но вы же очевидно живы.

И столь же очевидно мертв, разве нет? Вон же мой труп! Он есть? Есть.


Не стоит переоценивать смерть одной из ваших альтернатив

Но тогда и жизнь одной из альтернатив не стоит переоценивать?


Вашему внутреннему наблюдателю в сущности всё-равно, в каком альтернативном будущем он продолжит он наблюдать

И так же все равно — в каком не продолжит.

НЛО прилетело и опубликовало эту надпись здесь
Такой симметрии нет. Просто потому, что ни одна ваша альтернатива, кроме самых экзотических, никогда не увидит ваш труп.

Ну а с другой стороны, ни одна альтернатива, кроме самых экзотических, не увидит меня живым.


Жизнь ваших будущих альтернатив не стоит переоценивать в буддийском смысле

В том числе, не стоит переоценивать и то, что кто-то там в альтернативной вселенной выжил. Так?


Ну и действительно, всё равно — в каком не продолжит, имеет значение лишь в каком продолжит.

Почему это? Наоборот — важно в каком не продолжит. А в каком продолжит — не важно.


а вот жизнь со смертью — нет.

Чем несимметричны? И то и другое — просто физическое состояние.

НЛО прилетело и опубликовало эту надпись здесь
Как это не увидит вас живым? Все будут видеть вас живым, да хотя бы в зеркале.

И они же все будут видеть меня мертвым.


Кто-то там в альтернативном мире, в будущем, в прошлом жил или не жил. Вам, в данный момент, до всего этого далеко.

Эм. А при чем тут тогда вообще альтернативные миры?


Лишь в одном из этих состоянии есть наблюдатель, которого парят данные вопросы. Нет наблюдателя — нет вопросов.

Ну так это все точно так же работает и тогда, когда мир — один, инет никакого квантмеха. Разве нет?


одумайте лучше о том, что количество всех вопросов у вашей персоны обратно пропорционально количеству этих ваших персон.

Количество каких вопросов? Почему?

НЛО прилетело и опубликовало эту надпись здесь
Кто они? Я говорю про ваши альтернативы, про вас в других мирах. Никто из вас не увидит вас мёртвыми.

Если мир один, то меня мертвым тоже никто из моих альтернатив не увидит.


Нет. В мире без квантмеха после вашего исчезновения не останется информации, означающей ваше существование.

Вы опять непоследовательны. Почему вы обращаете внимание на информацию, означающую мою существование, но не обращаете внимание на информацию, означающую мою смерть?


То есть у вас есть два предиката — Жив и Мертв.
Если вы считаете Жив(х) истинным тогда, когда х жив хоть где-нибудь, то тогда и Мертв(х) вы необходимо должны считать истинным тогда, когда х мертв в каком-то из миров.


То есть точно так же как я всегда жив, я еще всегда и мертв. Иными словами — если в рамках ММИ я бессмертен, то отсюда сразу следует, что в рамках ММИ я никогда не рождался ни в одной из реальностей. Потому что если бы я родился, то я был бы мертв — что противоречит тому, что я жив.


Кроме того вы не обращаете внимание не следующий факт — реальность ветвится не во время смерти, а, по факту, до нее. Вот в одном мире вам падает на голову кирпич, а в другом — он остался лежать. Вы в мире с кирпичом (где вы умрете) — это один вы. А вы в мире без кирпича — это другой вы. И тот вы, что в мире с кирпичом — умерли. И какая вам разница что какой-то другой человек, с другой историей, остался жив?

НЛО прилетело и опубликовало эту надпись здесь
ричудливый мир, который всего один. Хорошо, что он фантастический, и думать о нём приходится только фантастам.

Почему фантастам? Это же наш мир, вполне реальный, данный нам в ощущениях.


А какая информация, по вашему, будет означать вашу смерть? Какая информация будет вообще иметь хоть какое-то значение для того, кто не существует?

Ну то есть в смерти ничего страшного нет, ведь тот кто умер осознать этого факта не может. Так?


По причине вышенаписанного не считаю эти понятия равнозначным.

По какой причине?


Жив это ноль и единица, а мёртв это их отсутствие.

Это же противоречиво.


реальность ветвится каждый раз, когда происходит квантовое событие, влияющее на вашу судьбу.

Но когда вы умираете этого не происходит. Т.к. сам процесс "вы умерли" нефизичен. Физична летящая в вас пуля (или не летящая). Физичен разрыв сосуда в мозгу (или не разрыв). И т.д.


точно так же, как мне и нет никакой разницы, что случится со мной через секунду.

Ну то есть в принципе, если бы вам предложили сто рублей за то, чтобы сыграть в русскую рулетку с шансом выиграть в 1%, то вы бы не отказались? Ведь вы ничего не теряете в любом случае, а вероятность выигрыша — она вот, есть.

НЛО прилетело и опубликовало эту надпись здесь
По причине вышенаписанного.

Там причины нет.


Не вижу причины так считать, укажите на неё.

У вас два предиката каждый из которых вы переносите на квантовомеханический случай по-своему.


Откуда вы знаете-то?

С точки зрения современной науки никакой души нет.


Ещё чего. Скорее всего я выживу с пулей в голове, что многовато для ста рублей.

Так это не вы, это кто-то другой, из другой реальности. А у вас пуля просто не вылетит. Верно же?

НЛО прилетело и опубликовало эту надпись здесь
«Русская рулетка» — это как-то грубо для мысленного эксперимента.

Допустим, пока вы спите ночью, кидается «квантовая монетка».
В 1 случае вам дают миллион долларов.
В 999999 случаях вас безболезненно убивают во сне (в теории, можно придумать массу способов превратить ваше тело в облако ионизированной плазмы, скажем, за миллисекунду).

Согласно ММИ, вы гарантированно проснетесь в одном из миров с миллионом долларов в кармане. Остальные ваши копии не почувствуют абсолютно ничего.

Надо соглашаться, нет?
НЛО прилетело и опубликовало эту надпись здесь
Ваше существование, и ваше не-существование это не равноценные понятия.

Почему?


Почему вы волнуетесь по поводу тех точек пространства, времени и мультиверса, где вы мертвы?

Ну по той же самой причине, по которой я волнуюсь за все те точки, где я жив? Хотя в этой точки вот он я — остывающий труп?


Ведь вы то живы здесь и сейчас

С другой стороны — здесь и сейчас я мертв. А жив — где-то там.


Почему вы для "жив" выбираете "здесь и сейчас", а для "мертв" где-то там? Почему не наоборот? Вот вы шли по улице, вам на голову упал кирпич и вы умерли. Какая вам разница, что где-то в другой реальности кирпич на вашу копию не падал? На вас-то он упал! Вы тут, в этой реальности. А не где-то "там"!


И как это относится к тому, что отсутствие информации и её наличие это вовсе не ноль и один?

При чем тут отсутствие информации и наличие?


С чего это? Я же не прекращу существовать со своей точки зрения, если выживу с огнестрельной травмой мозга?

Так вы не выживите, вы умрете. Выживет кто-то другой, в другой реальности, где и пистолет-то не выстрелил. Какое вам до этого другого дело?


Поэтому совершенно не факт, что мне должно повезти настолько сильно, что даже пуля не вылетит. Этого вам никто не обещал.

То, что пуля не вылетит — на порядок более вероятно чем то, что вы выживите с пулей в голове. Но это даже не важно, у нас ведь есть реальность где пуля не вылетела и этого для ваших рассуждений достаточно. Вы же выбираете для себя наилучшую возможную реальность, вне зависимости от ее вероятности. С-но, это та, где пуля не вылетела.


Ну нет, я бы не согласился. Мне кажется, что эмпирически непроверяемая теория всё равно не стоит такого риска.

Допустим, вы достоверно знаете, что ММИ верна, и один из 1000000 действительно гарантированно выжил с миллионом долларов. Что тогда?

НЛО прилетело и опубликовало эту надпись здесь
Потому что мёртвые не волнуются.

Тогда не понимаю, почему вы не хотите сыграть в вышеописанную рулетку. Ведь вин-вин ситуация — если вы выиграете, то выиграете, а если проиграете — то и волноваться некому будет. Так ведь?

НЛО прилетело и опубликовало эту надпись здесь
Вдруг я очнусь не с миллионом долларов а в цифровом аду, который устроил безумный сверхсильный ИИ из будущего, собравший всю информацию о когда-либо живших, и запустивший их симуляцию у себя в памят

Ну окей, уточним эксперимент. Если пуля вылетела — она гарантированно мгновенно вас убивает, и вы полностью прекращаете свое существование. Никаких цифровых (и не цифровых) адов. Тогда каков ваш ответ?

НЛО прилетело и опубликовало эту надпись здесь
Сама постановка вопроса исключает гарантию.

Допустим, у вас есть гарантия. Что тогда?

НЛО прилетело и опубликовало эту надпись здесь
Мне кажется что смысл как раз в принципиальной гарантии. Именно так многомиров и работает. Ты можешь узнать что было бы.
НЛО прилетело и опубликовало эту надпись здесь
Какая еще копия? Ничего не копируется.

Я вообще-то поддерживал ваше
Сама постановка вопроса исключает гарантию.
НЛО прилетело и опубликовало эту надпись здесь
В гарантии чего именно? Того, что очнусь именно я и с миллионом долларов?

Того, что если вы умерли — то умерли, все. Никаких копий нет.


А как это обеспечить?

Не важно. Допустим, обеспечили.
Или ваш ответ будет разным в зависимости от того, как обеспечивали?

Это глюк человеческого восприятия. Ну до кучи, к тому, что тервер не применяется для свершившихся событий, у них вероятность 1.

Дело в том, что «именно вы» это софт. Который был поставлен (развился) в тело готовое его воспринять. Так что грамотнее будет говорить о том, что «именно вы» скорее всего более здоровы чем больны, ибо изначально больные тела не дожили до появления «именно вы», или померли вскоре после этого. Так что, оду героическому сперматозоиду отменяем!
saaivs
если найти в природе (желательно в нашей окрестности) такой объект или систему объектов, который в зависимости от квантового события, вызывает надёжное уничтожение в локальной области всех потенциальных наблюдателей(но почему-то до сих пор этого не сделал, так же как ружье всё-время даёт почему-то даёт «осечку») — теорию тоже можно считать с высокой вероятностью доказанной. В общем, надо просто поискать «ружьё»)


Так найдено. Это… сама Вселенная.

На хабре была статья от Итана — если бы при рождении нашей Вселенной было бы на один протон больше, то Вселенная быстро сжалась бы обратно не успев породить даже звёзд.

А если бы при рождении нашей Вселенной было бы на один протон меньше, то Вселенная быстро расширилась бы и даже звёзды не смогли бы успеть образоваться.

Так что нам чрезвычайно повезло (без кавычек).

А еще возможно наш вакуум ненастоящий, и каждую секунду наша вселенная гибнет… Но мы этого не замечаем благодаря квантовому бессметрию

Так найдено. Это… сама Вселенная.


Пример интересный, но, боюсь, что онснован на большом количестве допущений. Никто не знает, что было бы в том или ином случае, т.к. выводы сделаны на основании той информации, о которой что-то знают, но никто даже понятия не имеет о том, что еще не известно, но так же надо принимать во внимание, поскольку эксперимент провести чтоб сравнить расчёты с реальностью как бы совсем без шансов.
Да, триггером для «рождения вселенной» во многих моделях является возможно явялется квантовое событие. Но точно никто не знает что именно и как именно всё происходило тогда, поскольку с трудом даются объяснения того что происходит прямо сейчас.
Обяснить квантовую механику на понятном человеку языке невозможно. Человек понимает только то к чему приспособлен эволюционно что воспринимают его органы чуств и для чего у него есть мысленые образы и понятия.
Обясните на понятном языке что такое спин. Или то же искривление пространства в ОТО.
Мы говорим не об интерпретациях в смысле как препод обясняет студентам а о ММИ претендующей на научную теорию. А значит речь не о понимании а о математическом апарате, предсказаниях и поджтверждение экспериментом.
Задача физики описать математически законы природы а не обяснить доярке что такое волновая функция

Объяснить КОНКРЕТНОМУ человеку, на по ПОНЯТНОМ ЕМУ языке — не всегда возможно, это верно. А в принципе для homo sapiens это не проблема и нужно просто больше фантазии и сообразительности. Подготовленный человеческий мозг хорошо переваривает любые абстрактные вещи :)
Физика занимается построением математической модели мира. И на микроуровне точно предсказать поведение частиц не получается, только вероятность. Волновой функции в реальности нет, это только модель, позволяющая считать вероятности. И модель не позволяет точно предсказать состояние кота. В реальности кот Шредингера либо жив, либо мертв, а альтернативный результат — это просто артефакт моделирования.
а как вы определяете что реально а что нет? Мы воспринимаем мир через органы чуств которые передаеют ограниченый обьем информации и то с искажениями. Наш глаз даже не может отличить пурпурный цвет от смеси красного с синим. Воспринимать квантовый уровень мы генетически не приспособлены вообще. Поэтому квантовая механика субьективна. Те же нарушения неравенств Белла говорят что обьективной реальности нет пока мы не провели измерение. Кот конечно не квантовый обьект это неуклюжая попытка Шредингера потролить квантовых механиков, но с точки зрения квантовой механики кот не жив и не мертв а находится в состоянии суперпозиции.
Или например корпускулярно волновой дуализм. Наблюдаем фотоэфект фотон частица наблюдаем двухщелевой эксперимент фотон (или кстати электрон) — волна. и что реально? какая частица на самом деле?
Есть теории что мы живем типа в матрице или например что вселенная на самом деле большой экран который нам показывают какие нибудь эксприментаторы. Я конечно в это не верю но технически это не противоречит никакой физике и в такой ситуации все были бы убеждены что мир реален в том числе и кот.

Реально то, что можно наблюдать или измерять. А физика — математическая модель, позволяющая предсказать результат будущих измерений. Состояние суперпозиции — тоже особенность математической модели.
Реальность — это философское понятие, не имеющее особого смысла в физике. Недавно была статья о том, что реальна любая непротиворечивая математическая конструкция. И что характерно, это теория может быть фальсифицирована: из нее следует, что колмогоровская сложность уравнений нашей вселенной близка к минимуму среди всех математических конструкций, которые содержат разум.
Наблюдать электроны например нельзя. Можно наблюдать только косвенные результаты что они существуют. И чем это отличается от косвенного налюдения суперпозиции?
А какие наблюдения суперпозиции? Как же «коллапс»? Наблюдаться то будет всегда конкретное состояние.
Суперпозиция это не физическое состояние в привычном смысле. так же как и электрон не шарик летающий вокруг атома (физики даже не знают размера электрона). И состояние зависит от наблюдателя. У вас есть электрон со спином в суперпозиции вверх и вниз. Вы провели измерение для вас произошел коллапс но для вашего ассистемна который еще не знает результатов вашего измерения вы находитесь теперь с электроном в запутаном состоянии с суперпозицией «спин вниз — вы знаете что он вниз» и «спин вверх вы знаете что он вверх». В этом заключается субективность квантовой механики — понятие реальности не имеет смысла до измерения.
Ну да, суперпозиция — не физическое состояние. И электрон — тоже некая абстракция. Мы можем провести измерение, результат которого интерпретируем как «спин электрона вверх» или «спин электрона вниз». И теория не позволяет предсказать результат этого измерения. Но что-то про ожидаемый результат сказать можно — вероятность разных исходов, корреляции с результатами других измерений. Вот для этой неопределенности результата придумали название.
Если вселенная это экран, то обязательно должны быть некие полученные в прошлом экспериментальные данные, неочевидные физикам тех лет, но очевидные сейчас. Грубо говоря, клад находят только когда ищут, но никогда не находили ранее (когда не искали).
Этот подход сильно напоминает мне аналогичные утверждения что Бог это простое обьяснение, а эволюция сложное.

По моему аналогично MWI это добавление почти бесконечного количества сущностей и новых вопросов которые просто занавесили одной табличкой.

(сам мнения по поводу квантмеха не имею)
Как уже MWI не добавляет ничего. Это в Копенгагенской интерпретации постулируется существование лангольеров
Вопрос интерпретации — это вопрос философии, а не физики.
Философов же «хлебом не корми, дай поспорить», т.к. всё равно нет способов проверить, кто из них прав («бритва» вовсе не является доказательством, примером тому все реальные убийства, замаскированные под несчастные случаи). Так что никакого «конца войны» тут нет и никогда не будет.
Давно подозревал, что Hawking на самом деле был философом гипотезы которого не возможно проверить.
Симулянты хреновы…
А тем временем где-то в параллельной вселенной учёные празднуют победу копенгагенской интерпретации.
Есть одна поправка — происходят не вообще все варианты, а все математически (т.е. и физически) допустимые варианты. Поэтому скорей всего, что не факт)
Есть одна поправка — происходят не вообще все варианты, а все математически (т.е. и физически)

Тут только важная оговорка в том, что "физически допустимо" в рамках квантмеха — это сильно шире, чем "физически допустимо" в рамках классической физики.

в любом случае «квантмех» существует в рамах «математики»… а «физика» упомянута лишь как ее частный случай.
НЛО прилетело и опубликовало эту надпись здесь

Это очень интересный вопрос, но надо честно признаться: мы не знаем, и нет даже идей. Нет ни понимания, что такое сознание, и соответственно, нет понимания, что такое "сейчас" в физике

Говорят, что это связано с особенностями работы мозга. Наш мозг все время занимается предсказанием событий в будущем и корректированием работы в зависимости от результатов (чтобы успевать предугадывать действия хищников, например). Это задает причинно-следственную связь между предсказанием и данными реального мира и определяет некоторый промежуток времени, на который нам нужно предсказывать. Этот промежуток и создает у нас восприятие движения во времени.
НЛО прилетело и опубликовало эту надпись здесь
Мы даже не знаем что такое «восприятие» (Qualia)
(((
А представьте, кроме 4D объекта ещё есть волновая функция, которая описывает состояние 4D объекта.

Например, если вместо 4D возьмем прямую 1D(«наш мир») и поместим её на линейчатую поверхность, например, на геликоид или гиперболоид вращения («волновая функция»). То если задать некторый трек перхода от одной образующей прямой к другой на этой поверхности — и считать это «ответвлением» — это и будет своего рода «эволюция». Несмотря на то, что прямая всегда прямая(как и «4D» всегда «4D») и кажется статичной тем, кто на ней, но по факту она, как видим, просто перемещается по некоторой поверхности и вполне себе «динамична».
3D срез мозга (который моделирует сознание) просматривает 3D срез. 4D мозг просматривает 4D временной конус прошлого. Будущего не видит, т. к. время течёт в одну сторону, 3 закон термодинамики, принцип причинности, все дела. Если не учитывать квантмех, следующая фотография — результат какой-то функции от предыдущего.
НЛО прилетело и опубликовало эту надпись здесь
Да нет же, сознание не просматривает срез. В каждом моменте одно какое-то 3D сознание воспринимает одну 3D сцену. Просто нету одного 4D сознания. Постоянно в мозгу что-то происходит. Каждую секунду сознание другое. Через несколько метров от меня другой человек, он видит мир немного с другой точки и немного по другому. Так же само и через несколько минут от меня другой человек (хоть и очень похож на меня, та же ДНК и т.д.). Все моменты времени одинаково реальны.
НЛО прилетело и опубликовало эту надпись здесь
Это не моя идея. Я очень давно это читал еще школьником в популяризаторской книге.

От того что человек находится, например, в Токио, другие города не менее реальны. Просто в других городах его нет. Так же само и другие моменты времени. Меня нет в 2020 и не будет. Там будет человек почти как я, но немного другой (если доживу). Потому я и не могу р-раз и наблюдать 2020. А тот человек может и делает это. 2020 год такой же реальный, как и город в котором меня нету.
Тогда и вчера вас не было? И секунду назад? Отсюда вопрос: «Что есть Я?»

Эта позиция имеет право на жизнь (хотя с ней мало кто согласен), называется Presentism. Правда, он несовместим с теорией относительности

Как это не совместим? Это не какая-то философия, а банальный факт. Мозг меняется.

В теории относительности нет общего "сейчас"

Движением частей мозга относительно друг друга можно пренебречь. Для всего мозга будет очень точное общее «сейчас».

Да, но презентизм ведь о мире вообще, что существует только "настоящее"

Да, существует только настоящее, но оно локально. Вы пытаетесь сделать снимок всего пространства-времени и оперировать набором таких снимков. Но смысл не в том, есть ли снимок сейчас, снимок вчера и снимок завтра, а в том, что нет прошлого, нет будущего, есть только частицы и взаимодействия, которые происходят сейчас в прострастве-времени.

Значит у вас еще более экстремальная версия презентизма

Да, и у неё нет проблем с теорией относительности. Зачем тогда усложнять и умножать сущности?
Видел когда-то как изометрические преобразования в пространстве Минковского (преобразования Лоренца) соответствуют конформным преобразованиям бесконечно удаленной и бесконечно будущей (прошлой) двумерной сферы. А если брать связную часть группы Лоренца, то хватит и одной сферы.
Ну и похоже что квантовые поля в пространстве Минковского соответствуют квантовым полям на тех сферах. Пусть меня поправят если не так, но выходит что всё квантовое поле в 4D можно восстановить по двум квантовым полям на двух 2D сферах.
То есть здесь не совсем презентизм, а какой-то пастизм-футуризм: хватит только двух моментов времени что-бы описать всю ту чертовщину которая происходит.
Точечная частица в 3D — это точка, а в 4D — кривая. С человеком, конечно же, не так все просто. Но можно назвать некоторый паттерн расположения частиц — «я». Частицы будут добавляться в него и убираться, но паттерн будет сохраняться и меняться не очень быстро.

Но ветка началась с
Почему тогда сознание постепенно просматривает только 3D срез 4D пирога?
Вот с этим вот я не согласился. 3D срез 4D пирога просматривает 3D срез 4D паттерна, которым и есть «я».
НЛО прилетело и опубликовало эту надпись здесь
Кирпич тоже меняется от взаимодействия с миром, значит он тоже «просматривает», просто из-за его конструкции он мало что может.
НЛО прилетело и опубликовало эту надпись здесь
Нету никакого панпсихизма. Нету в кирпича психики. Вы писали
Почему тогда сознание постепенно просматривает только 3D срез 4D пирога?
Любой объект (не важно что там с сознанием) — четырехмерный. И в каждый конкретный момент времени (когда наш объект существует), есть только 3D срез этого объекта. Так вот этот 3D срез «просматривает» 3D срез мира. Взаимодействует с ним. Не нужно для этого никакое сознание.
НЛО прилетело и опубликовало эту надпись здесь
ползущей по перфоленте
И этого нет. Ничего не ползет. Все моменты времени одинаково реальны. У меня сейчас в комнате в момент 1:14:50 есть 3D срез мозга, он просматривает 3D срез комнаты в момент 1:14:50 (на несколько наносекунд раньше, пока свет летит). В момент 1:14:51 есть другой 3D срез мозга, он просматривает другой (почти такой же) срез комнаты в момент 1:14:51, сам мозг немного отличающийся, который помнит себя секундой раньше. Каждый моментальный снимок мозга помнит другой снимок мозга чуть раньше. Это как связанный список, где связи — память. От того и ощущение что что-то куда-то идет. Время не идет. И это не моя гипотеза. Это давно известный банальный факт.
НЛО прилетело и опубликовало эту надпись здесь
Блокчейн, да?
Нет. Самая обычная человеческая память.
Это значит, что если взять все таким образом связанные 3д срезы мозга, и записать на какой-то носитель, то одно лишь существование такого носителя будет означать, что существует такое сознание, которое ощущает ход времени по всем этим кадрам последовательно.
Типа того. Мне кажется что это очевидно.
НЛО прилетело и опубликовало эту надпись здесь
НЛО прилетело и опубликовало эту надпись здесь
НЛО прилетело и опубликовало эту надпись здесь
Не всего лишь. Всего лишь цепочка может быть просто записана где-то, а вычисление это то что произошло.
НЛО прилетело и опубликовало эту надпись здесь
В СТО пространственноподобные и времениподобные кривые отличаются. Если что-то записано на пространственноподобном носителе — это не произошло, если на времениподобном — это произошло.
НЛО прилетело и опубликовало эту надпись здесь
Отличить можно. Если загружаешь в интерпретатор знаки последовательно, то четверка появится сама. А если пишешь на бумаге, то не появится пока сам не напишешь.
НЛО прилетело и опубликовало эту надпись здесь
не большим образом, чем в вольфрамовой нити “зашифрован” свет у лампочки накаливания…
НЛО прилетело и опубликовало эту надпись здесь
Можно рассчитать что будет происходить в каком-то другом месте.

Это напомнило мне о размышлениях о событиях на земле. Большинство энергии земля получает от солнца. Оно приводит в движение почти все. Оно — источник тепловой случайности, которая ответственна за бытовую случайность. Не значит ли это что события происходящие на земле на самом деле происходят на солнце? Что вообще означает «событие именно здесь»? Выходит что языческие боги — более рациональный взгляд на мир чем авраамистические религии.
Не подав в данное время на эту нить заданное напряжение, не учитывая температуры нити на этот самый момент времени, мы не получим излучения с конкретными характеристиками.

В саму нить может быть зашифровано что угодно, но только реальные взаимодействия в данный момент времени “извлекают” из нити строго определённое поведение.

P.S. А могли бы, скажем, не светить при помощи нити, а использовать её как пружину…
Значит, что в числе Pi можно рано или поздно найти множество таких длинных участков цифр, каждый из которых будет исчерпывающе определять каждый 3д срез каждого мыслящего существа во Вселенной?
Думаю что да. Можно найти. А можно и самому записать (запрограммировать в компьютере). Но пока этого не сделано — этого нет.
НЛО прилетело и опубликовало эту надпись здесь
Итак, раз сознание есть, и сознание всего-лишь взаимодействие, то любой набор взаимодействующих частиц имеет сознание.

Если вода существует, и она есть всего лишь жидкость, это не значит, что всё, что жидкость — это вода.
НЛО прилетело и опубликовало эту надпись здесь
Ну так и сознание это не любое взаимодействие, а строго определенного типа.
НЛО прилетело и опубликовало эту надпись здесь
Но эти взаимодействия не определяют воду — нам нужна динамика на количестве молекул, внешние условия и прочее. То же самое и с сознанием =)
НЛО прилетело и опубликовало эту надпись здесь
если кирпич и вода оказывает одинаковое давление на опору, это не значит, что они оба жидкие… Поправьте меня, если я ошибаюсь…
НЛО прилетело и опубликовало эту надпись здесь
но почему вы решили, что сознание определяется “просматриванием среза”?
НЛО прилетело и опубликовало эту надпись здесь
Так в том то и дело что не одинаково. Кирпич очень слабо и очень просто взаимодействует, по сравнению с мозгом. Зефир и рыба взаимодействуют со вкусовыми рецепторами. Зефир это рыба? Вся еда это рыба?
НЛО прилетело и опубликовало эту надпись здесь
Между кирпичом и мозгом также само. На одном уровне разница несущественна, а на другом существенна. Но сознанием кирпич не обладает ни в каком смысле.
НЛО прилетело и опубликовало эту надпись здесь
Там «просматривает» в кавычках именно потому.
Существование этих ветвей неизбежно следует из формул квантовой механики
Ну ведь не следует же, лукавит автор. Точнее, «следует» даже меньше, чем из уравнения окружности следует существование окружностей с отрицательным радиусом.
НЛО прилетело и опубликовало эту надпись здесь
Тогда что это такое?
НЛО прилетело и опубликовало эту надпись здесь
И?
НЛО прилетело и опубликовало эту надпись здесь
А разве я писал -x^y или x^-y?
НЛО прилетело и опубликовало эту надпись здесь
Ключевое слово здесь «был»)
А он никуда и не девался. Вы всё ещё можете подставлять в формулу окружности отрицательный радиус. И получать окружность. Правда уже с положительным радиусом. Фишка как раз в том, что не всё, что корректно с точки зрения математики имеет смысл.
Фишка в том, что всё, что корректно с точки зрения математики — только оно и имеет смысл. Тот факт, что конкретный субъект что-то не состоянии понять и осмыслить — это частные проблемы конкретного индивида.
Отлично. Жду от вас построения окружности радиуса -1. С точки зрения математики здесь всё корректно.
Ответить на вопрос — значит согласиться с его постановкой :)

С точки зрения математики здесь всё корректно

Вы не задумывались над тем, чтобы немного подтянуть свои знания математике?
Здесь проблема в интерпретации понятия «радиус». Если считать что это то самое R в уравнении, тогда он может быть отрицательным. Если считать что это расстояние до центра, то не может. Обычно считают второе (расстояние до центра).
Здесь проблема в интерпретации понятия «радиус»

Если у вас (или у тех кто там как то «считает») проблема в интерпритации этого понятия, то в математике как в научной дициплине такой проблемы нет — аксиоматика метрических пространств вполне неплохо проработана.

Зачем же перекладывать с больной головы на здоровую?

Даже Википедия и та выстраивает вполне однозначную цепочку по связке «радиус-математика»:
ru.wikipedia.org/wiki/Радиус
ru.wikipedia.org/wiki/Длина
ru.wikipedia.org/wiki/Расстояние
https://ru.wikipedia.org/wiki/Метрика_(математика)
ru.wikipedia.org/wiki/Метрическое_пространство
Именно потому я написал
Обычно считают второе (расстояние до центра).
А оно не может быть отрицательным. Но ведь человек считал что это R в уравнении, в том и ошибка. Но если говорить «построй окружность с отрицательным R в уравнении», то нету никаких проблем такую построить.
Человек (у которого в отличии от девочки есть имя) не говорил такого. Человек (раз уж вы и saaivs не любите помнить с чего начался разговор) говорил, что уравнение окружности, в которое радиус входит не напрямую, а в виде собственного квадрата, допускает существование окружностей с отрицательным радиусом. Хотя такие окружности существовать не могут просто в силу определения. Того самого, геометрического места точек, равноудалённых от данной.
Уравнение в окружности в декартовых координтах — это лишь формальный способ записи. Один из. Модель. Посмотрите, как выглядит то же уравнение в полярной системе координат, где R будет на неотрицательной оси по определению и там такой трюк не прокатит.

Математические модели бывают удачные и не очень. Но от выбора модели абстрактные свойства объекта не меняются. Просто на моделях их удобней изучать.

Чтоб допустить существование «существование окружностей с отрицательным радиусом» — нужно сначало такую сущность определить.

А что у вас получилось? У вас хвост виляет собакой.
Я выбрал простой и понятный пример, чтобы проиллюстрировать свою мысль. Ну знаете, как при популяризации науки что-то излагают на пальцах при помощи аналогий. Вот я и выбрал пример, который понять может даже школьник. А вы поступили как классический альтернативщик — взяли упрощённую аналогию и сделали из неё далеко идущие выводы… «А вот если заменить декартовы координаты на полярные...» Но мой пример был не для того, чтобы в нём что-то менять. Он был ровно для того, для чего был сказан. Я не пытался вывести теорию отрицательных окружностей.
А хвост собакой виляет не у меня, а у тех, кто пытается сказать на основании аналогичных умозаключений, что у ММИ есть какие-то преимущества перед другими интерпретациями.
у ММИ есть какие-то преимущества перед другими интерпретациями


Никто не утверждает, что она истинна. Просто пока что очевидным является тот факт, что она более минималистична, чем все известные другие. При прочих равных это уже немало и её наблюдаемое приемущество исключительно в этом. А эксперименты, вроде «квантового бессмертия» пытаются придать ей лишь черты фальсифицируемости, насколько вообще уместо об этом говорить.
Это не преимущество, а такое же жонглирование словами. В одних случаях приходится вводить дополнительные сущности, чтобы описать наблюдаемую вселенную, а в других ничего не вводить, но принять на веру существование огромного пласта реальности, с которым мы никак не можем взаимодействовать. Я даже не знаю как это можно сравнивать.
Это как эфир в преобразованиях Лоренца. Вселенная так устроена, что вы не можете его обнаружить. Но он есть, вы верьте. :)
В одних случаях приходится вводить дополнительные сущности,… а в других ничего не вводить, но принять на веру...

Вот в этом вся и суть. В одних надо вводить, а в других не надо. В одном случае вы что-то сами придумываете, а в других это что-то является логическим следствием — разница принципиальна. Поэтому чем меньше надо вводить базовых понятий — тем лучше. Принять «на веру» можно только базовые постулаты — остальное придётся принимать автоматически независимо от вашей интуиции и отношения к полученным результатам. Либо находите ошибки, либо принимайте результаты как есть, какими бы они ни бы ли. Если вы этого не понимаете — это объясняет почему у вас тогда такой диссонанс.
Предлагаю принять Бога в качестве базового постулата. Получим предельно простую теорию всего с нулевым математическим багажом и 100% точностью объясняющую мир. Из недостатков — отсутствие предсказательности теории. Ещё самого Бога никаким способом обнаружить нельзя. Но в рамках теории его существование где-то там следует из существования мира.
2 тыс. лет назад это план успешно претворили в жизнь. Потом ещё несколько раз. Вообще схема вполне рабочая.
А я что говорю, гениально! Чего только отказались, все новые модели явно менее совершенны.
Уравнения квантовой механики — тоже лишь формальный способ записи, математическая модель, позволяющая считать вероятности.
Но вообще я неправ. Позвльте принести свои извинения.
Просто когда смешивают понятия вне контекста то и получаются такие диалоги. Дело в том, что «отрицательное занчения компоненты радиуса в уравнении окружности» совсем не то же самое, что «построить окружность с отрицательным радиусом»… в одном случае будет вполне себе нормальное ГМТ, а в другом белеберда. Из уравнение окружности не следует, что радиус может быть отрицательным просто по причине того, что при отрицательных значениях он перестаёт им быть практически по определению. Задайте вы вопрос в виде: Постройте ГМТ по уравнению окружности при R=-1, это же не вызвало бы диссонанса, верно? Окружностей с отрицательным радиусом не бывает. А построить окружность по уравнению при заданных значениях параметров — можно. Опять же, уравнение уравнению рознь. Возьмите полярные координаты и история будет совсем другая — там вопрос об отрицательном значении радиуса уже вообще потеряет всякий смысл.
Мило. Вот так люди посылают других учиться, а сами элементарных вещей понять не могут. Из уравнения окружности следует только одно: В него можно подставить бессмысленное с точки зрения геометрии значение, но оно посчитается и останется корректным (с точки зрения математики). Если что-то имеет смысл, его можно посчитать соответствующей формулой. Если что-то можно посчитать это ещё не значит, что в этом есть смысл. Что тут не понятного?
Если уравнения квантовой механики допускают ММИ, это ещё ни разу не значит, что так оно и есть.
Ничего вы не поняли.
НЛО прилетело и опубликовало эту надпись здесь
В него можно подставить бессмысленное с точки зрения геометрии значение, но оно посчитается и останется корректным (с точки зрения математики)


Параметры в уравнении имеют смысл только из области определения. Выполняя же безсмысленные действия и результат получается бессмысленный. Но противоречия не возникает. Уравнения могут предсказывать значения параметров только на области определения, иначе и получится билеберда вроде «отрицательного радиуса». Сами уравнения же область определения не всегда задают. Иначе можно сказать, что радиус вообще может быть не только отрицательный, но и «комплексный», например. Уравнение «стерпит». Выражение «a + 1= b», ничего не говорит о природе переменных «а» или «b» и из него само по себе вне контектса ничего не следует.

Уравнения квантмеха не просто допускают концепцию MMI, а логически из них следует. Это не то, во что можно верить или не верить. Если есть претензии к этому утверждению, то надо смотреть на базовые постулаты или на вывод и искать «нестыковки» там.
Сами уравнения же область определения не всегда задают.
Не не всегда, а никогда. Сами же написали:
Уравнения могут предсказывать значения параметров только на области определения
и из него само по себе вне контектса ничего не следует.
Но тут же вы начинаете новый абзац, в котором у вас из уравнений КМ выводится её определение, её контекст. Как?
Не не всегда, а никогда.

Не совсем так. например y = 1/x где область определения — все действительный числа, то x = 0 выпадает из-за смысла математической операции деления, что заданно как раз уравнением.

из уравнений КМ выводится её определение, её контекст. Как?


Можете точнее процитировать мою фразу, которую вы так интерпритируете. Я не пойму что вы имеете ввиду.
Просьба к публике, читавшей мою предыдущую статью поучаствовать в коротком опросе на тему сознания и Qualia

Добавьте в форме для каждого вопроса «другое», где можно написать своё мнение.
Спасибо за интересную статью. Я согласен с тем, что классическая копенгагенская интерпретация имеет кучу проблем, самой очевидной из которых является вопрос о том, чем с физической точки зрения является сам процесс измерения. Но всё же у меня имеется ряд вопросов касательного того, каким образом многомировая интерпретация объясняет те или иные эксперименты. Если данная интерпретация является последовательной и логически непротиворечивой, то она должна объяснять результаты всех квантовых экспериментов как минимум не хуже любой другой интерпретации

1) Каким всё-таки образом многомировая интерпретация объясняет «расщепление» вселенных, в случае если вероятности исхода эксперимента не 50/50, как в случае классического эксперимента с двумя щелями а, например 99/1 или 60/40? Если предполагать, что каждому собственному вектору состояния измеряемой системы соответствует лишь одна параллельная вселенная, то из многомировой интерпретации будет следовать, что одни вселенные, скажем так, более «реальные», чем другие. Если же предполагать, что расщепление будет идти по принципу 99 вселенных с одним исходом и 1 вселенная с другим исходом, то совершенно непонятно, из каких общих принципов это следует

2) Из первого вопроса сразу же следует второй. Каким образом многомировая интерпретация объясняет прецессию спина электрона в магнитном поле, когда динамически изменяются коэффициенты суперпозиции? Если считать, что каждое собственное значение вектора состояния спина электрона соответствует значению спина в отдельной вселенной, то получается, что колебания коэффициентов суперпозиции делают то одну вселенную более реальной, то другую? Причём если колебания коэффициентов происходят в интервале от 0 до 1 включительно, то при достижении одного из коэффициентов нуля, то одна вселенная, то другая на мгновение пропадают, а потом материализуются вновь?

3) Может этот вопрос покажется банальным или даже глупым. Но если так, то ответить на него будет наиболее просто. Почему с точки зрения многомировой интерпретации вообще существует принцип неопределённости? Каким образом он согласуется с этой интерпретацией?

4) Третий вопрос примерно аналогичен второму. Почему с точки зрения многомировой интерпретации вообще происходит интерференция на двух щелях, если отсутствует информация о траектории движения микрочастицы?

5) Каким образом многомировая интерпретация объясняет и описывает существование виртуальных частиц и виртуальных переходов? Каким образом многомировая интерпретация вообще объясняет существование энергетических уровней в атомах со всеми их узловыми точками и дискретными переходами?

6) Каким образом многомировая интерпретация объясняет квантовые биения при релаксации электронов в атомах с высших энергетических уровней?

7) Что происходит при распаде свободного нейтрона с точки зрения многомировой интерпретации? Происходит постоянное «расщепление» вселенных на вселенную с распавшимся и не распавшимся нейтроном? С какой скорость «генерируются» эти альтернативные ветки? В случае распада различных ядерных резонансов это «расщепление» происходит быстрее?

8) Каким образом многомировая интерпретация объясняет эксперимент Пфлигора-Менделя (Pfleegor-Mandel experiment), когда один фотон фактически испускается двумя лазерами одновременно?

9) Каким образом многомировая интерпретация объясняет такие эксперименты как двухщелевой эксперимент с отложенным выбором, квантовый ластик и квантовый ластик с отложенным выбором? В экспериментах с квантовым ластиком когерентность вообще может быть восстановлена

10) Каким образом многомировая интерпретация объясняет эксперимент с ТРЕМЯ щелями, в которых так называемые «невозможные» траектории микрочастицы дают вклад в интерференционную картину? Логичным продолжением этого вопроса будет вопрос о том, каким образом многомировая интерпретация согласуется с формализмом интегралов по траекториям, согласно которому микрочастица фактически проходит по всем возможным траектория одновременно, но эти траектории имеют различный вес и траектории имеют тем больший вес, чем они ближе к классическим. Можно, конечно, сказать, что в каждой из альтернативных вселенных своя частица проходит по своей траектории, но в таком случае опять выходит, что одни вселенные получаются реальнее других. И совершенно непонятно, почему вообще происходит интерференция при «взаимодействии» этих траекторий

11) С точки зрения многомировой интерпретации может происходить так называемая «склейка» альтернативных вселенных, при условии что одно и тоже состояние системы может быть достигнуто двумя или более различными путями. Каким образом происходит этот процесс склейки и чем он является с физической точки зрения?

12) С точки зрения многомировой интерпретации могут существовать такие вселенные, в которых электрон или фотон всегда проходят через правую или всегда через левую щель. Могут быть такие, где в 90% случаев частицы будут проходить через левую или через правую щель и в 10% случаев через другую. Могут быть такие, где свободные нейтроны вообще не распадаются и так далее. С одной стороны, можно от этого откреститься и сказать, что да, с мизерной вероятностью такие вселенные в рамках многомировой интерпретации действительно могут существовать. Но где гарантия, что мы находимся именно в одной из наиболее вероятных вселенных? С одной стороны кажется совершенно логичным, что в нашей вселенной частицы проходят примерно в 50% случаев через левую щель и примерно в 50% случаев через правую, как и должно быть во вселенной, имеющей наибольшую вероятность. Но где гарантия, что именно наши вычисления вероятностей исходов экспериментов являются каноническими? Может наиболее вероятно, что по каким то причинам в самой общей вселенной электроны больше тяготеют к левой щели, чем к правой, а мы живём в менее вероятной вселенной, которая лишь нам кажется наиболее канонической? Многомировая интерпретация хоть как то разрешает подобные вопросы?

P.S. Буквально вчера смотрел на youtube канал пользователя LightCone, где товарищ поставил себе целью достаточно доступно и одновременно с этим достаточно строго изложить основы квантовой механики. Видно, что автор канала неплохо разбирается в предмете. Так вот, он там в одном из своих видео с такой же железобетонной уверенность утверждает, что никакая интерпретация кроме копенгагенской даже логически невозможна, так как с его позиции альтернативные интерпретации являются внутренне противоречивыми. А люди, утверждающие обратное просто нормально не разобрались в предмете. Если почитать и послушать современных крупных учёных в области квантовой теории, то складывается такое впечатление, что тот или иной человек как правило придерживается той интерпретации, которая нравится больше лично ему. Хотя аргументация в поддержку той или иной точки зрения, естественно, присутствует

P.P.S. Не стоит думать, что изложенные мной вопросы говорят о том, что я придерживаюсь копенгагенской интерпретации. Относительно неё можно также задавать множество неудобных вопросов, на которые не существует ответа в рамках копенгагенской интерпретации

P.P.P.S. И ещё вот что хочу добавить. Как я понимаю, автор данной статьи интересуется философией сознания. При этом он утверждает, что копенгагенская интерпретация проигрывает многомировой по причине введения постулата об измерении и коллапсе волновой функции, которого нет в многомировой интерпретации. И якобы по этой причине многомировая интерпретация выигрывает у копенгагенской с точке зрения минималистичности теории. Хочу заметить, что с точки зрения классического материализма само понятия феноменального сознания, которым интересуется автор, является избыточным конструктом. Собственно, такие философы как Даниэль Деннет и Пол и Патриция Черчленды пошли по этому пути и в принципе отрицают наличие сознания. С чем автор не соглашается в одной из своих статей, посвящённых сознанию. Но в рассуждениях по физике вы по сути применили ту же логику, против которой вы выступаете в вопросах исследования сознания
1,12. Как раз это объясняет правило Борна
3. С точки зрения MWI частиц вообще нет, есть только волны
4. См 2,3
11. Нет, насколько я понимаю
7. По мере того, как наблюдатели наблюдают результаты этого процесса, то есть не быстрее скорости света. Если какой то наблюдатель не заметил распада нейтрона, то ему пофик

2,5,6,8,9,19. также как квантовая механика. Вообще объяснение ВСЕХ экспериментов со сколько угодно сложными интерференциями в MWI на редкость тривиально, так как до декогеренции есть просто рассчет волновой функции по правилам квантовой механики

Что именно объясняет правило Борна в контексте многомировой интерпретации? Можно получить более или менее развёрнутый ответ? Если принять одновременно и многомировую интерпретацию и правило Борна, то выходит, что одни вселенные в каком то смысле реальнее других. Прецессия спина электрона в магнитном поле только обостряет этот вопрос. Пока что мне непонятно, каким образом многомировая интерпретация решает эту проблему

Опять же, совершенно непонятно каким образом утверждение о том, что частиц нет, а есть только волны решает проблему принципа неопределённости. Каким например образом то, что все частицы якобы являются только волнами разрешает неопределённость энергия-время?

Что касается декогеренции как универсального объяснения, то эксперимент с квантовым ластиком с отложенным выбором доказывает, что когерентность может быть восстановлена. И ключевым условием того, будет частица проявлять волновые или корпускулярные свойства, является наличие или отсутствие информации о траектории частицы. Каким образом многомировая интерпретация объясняет этот вопрос?

Что касается 11 пункта, то такое понятие как склейка в многомировой интерпретации есть. Можете также поискать, например, такое понятие как множественные истории

Что касается последнего ответа про пункты 2,5,6,8,9 и так далее. Можно подробно разобрать для начала хотя бы один пункт с точки зрения многомировой интерпретации и показать, что никаких противоречий с экспериментальными данными не возникает? А не просто декларировать, что всё нормально
Что именно объясняет правило Борна в контексте многомировой интерпретации? Можно получить более или менее развёрнутый ответ?


Оно объясняет то, что вы статистически наблюдате наиболее «вероятные» события. Без правила Борна это было проблемой для MWI, так как она детерминирована (и не понятно, что такое «вероятность» в ней). То есть когда вы смотрите на нейтрон, то есть ветки где он не распадется и через год. Но для вашего сознания вероятность попасть в эту ветку очень мала.

Прецессия спина электрона в магнитном поле только обостряет этот вопрос. Пока что мне непонятно, каким образом многомировая интерпретация решает эту проблему


Непонятно в чем проблема

Что касается последнего ответа про пункты 2,5,6,8,9 и так далее. Можно подробно разобрать для начала хотя бы один пункт с точки зрения многомировой интерпретации и показать, что никаких противоречий с экспериментальными данными не возникает? А не просто декларировать, что всё нормально


Вначале объясните где вы видите проблему, так как все экспериментальные предсказания MWI совпадают с предсказаниями квантовой механики. Напротив, очень тонкий вопрос, можно ли сфальсифицировать это и перевести MWI из интерпретации в теории.

является наличие или отсутствие информации о траектории частицы. Каким образом многомировая интерпретация объясняет этот вопрос?


Какой траектории? Какой частицы? В MWI нет траекторий и частиц. Есть волновая функция.
Если принять одновременно и многомировую интерпретацию и правило Борна, то выходит, что одни вселенные в каком то смысле реальнее других.

Именно так. И модуль коэффициента при разложении по соответствующему вектору — отражает, насколько данный "мир" реален.


Пока что мне непонятно, каким образом многомировая интерпретация решает эту проблему

Ну у вас есть два "мира" (состояния) и "реальность" туда-сюда плавно перетекает делая попеременно реальным то один мир, то другой.


Каким например образом то, что все частицы якобы являются только волнами разрешает неопределённость энергия-время?

А ее не надо разрешать. Неопределенность есть, т.к. никакой процесс в квантмехе не происходит мгновенно — у-е Шредингера это диффур, все процессы непрерывны. Более того — нет вообще самого понятия какого-либо "процесса" т.к. захватив соседние состояния этот же процесс будет уже немного другим (так же как с котом Шредингера а вас-то на самом деле кот не один — их несчетное число, отличающихся друг от друга позицией отдельного атома, например) и никакого разумного способа разделить объекты — нет. На самом деле, если у нас есть атом, то мы даже не можем каким-то разумным образом сказать, где заканчивается состояние одного электрона и начинается состояние следующего.


Каким образом многомировая интерпретация объясняет этот вопрос?

Частица всегда ведет себя как волна и никогда — как частица. Так что и проблем нет.


Что касается 11 пункта, то такое понятие как склейка в многомировой интерпретации есть.

Чтобы произошла склейка, надо чтобы после склейки нельзя было восстановить разницу в историях. То есть она если и возможна — только для очень малых систем.
Ну и с-но пробелмы никакой нет, на примере кота — был живой кот (некотоырй волновой пакет), вы начали эксперимент, волновой пакет разделился на два. Точно так же два пакета могут слиться в один.


2.


получается, что колебания коэффициентов суперпозиции делают то одну вселенную более реальной, то другую?

Все так.


то одна вселенная, то другая на мгновение пропадают, а потом материализуются вновь?

Не так. Все состояния существуют в пр-ве состояний всегда, меняется только их "реальность".
Вся вселенная — просто набор всех возможных состояний, каждому из которых в каждый из моментов времени сопоставлен некоторый "коэффициент реальности" (то есть у нас скалярное поле определенное на пр-ве состояний). У-е Шредингера, с-но, описывает, динамику этого скалярного поля — то есть то, как "реальность" перетекает из одного состояния в другие. То есть вероятность обнаружить систему в первом состоянии — постепенно падает, а вероятность обнаружить систему во втором состоянии — постепенно растет. При этом нет никакого скачка, когда система бац! и перешла из одного в другое.


Почему с точки зрения многомировой интерпретации вообще существует принцип неопределённости? Каким образом он согласуется с этой интерпретацией?

Из-за неравенства Коши-Буняковского, то есть из-за базовых св-в геометрии пр-ва состояний (ну это в любой интерпретации)


Каким образом многомировая интерпретация вообще объясняет существование энергетических уровней в атомах со всеми их узловыми точками и дискретными переходами?

В квантовой механике не бывает дискретных переходов, т.к. у-е Шредингера — это диффур и его решение — непрерывная ф-я.
То, что вы наблюдаете как "дискретный переход" — это плавное перетекание "реальности" из одного состояния в другое.


Происходит постоянное «расщепление» вселенных на вселенную с распавшимся и не распавшимся нейтроном? С какой скорость «генерируются» эти альтернативные ветки? В случае распада различных ядерных резонансов это «расщепление» происходит быстрее?

Все зависит от того, как описывается система в общем. Если система подвержена эволюции — то она сама будет некоторым "волновым пакетом" состояний ("компактный" набор состояний, в которых "много" реальности), в которых событие не произошло, этот "пакет" будет оставлять за собой (истираясь) след в виде непрерывного гребня из состояний, в которых событие произошло. Этакий вытекающий ручей. Если же она "стабильна" то просто будет "реальность" перетекать из одного состояния в другое.


8,9,10 — везде и всегда все объекты ведут себя как волна, с-но все интерференционные опыты объясняются тривиально


12) С точки зрения многомировой интерпретации могут существовать такие вселенные

Это проблема квантмеха в принципе, не интерпретации. Всегда может оказаться так, что наши наблюдения — это статистический казус и НА САМОМ ДЕЛЕ вселенная ведет себя совершенно иначе.
К счастью, вероятность такой ситуации сильно, очень сильно ниже вероятности того, что в следующее мгновения Земля исчезнет.

Всегда может оказаться так, что наши наблюдения — это статистический казус и НА САМОМ ДЕЛЕ вселенная ведет себя совершенно иначе.
К счастью, вероятность такой ситуации сильно, очень сильно ниже вероятности того, что в следующее мгновения Земля исчезнет.


Здесь есть кстати очень забавная штука
Вы помните про «Мозг Больцмана» — случайным образом возникщий в бесконечной вселенной мозг со всеми воспоминаниями, который в следующую секунду разрушится. В нашей вселенной существование такого мозга практически невероятно (но если вселенная бесконечна то они возникают).

Таких наблюдателей называют «freak observers»
etheses.lse.ac.uk/2642/1/U615591.pdf

Однако в «неудачной» вселенной, где жизни не возникнуть, только такие и есть!
«Мозг Больцмана» — попытка применить математическую абстракцию (а как получить бесконечность? что надо с ней математически сделать чтобы она стала конечной?) к реальному физическому миру.
Давайте разберём всё это в упрощённом виде на примере спина электрона. Спин электрона может принимать значение либо +1/2, либо -1/2 по направлению определённой оси. Теперь предположим, что мы берём случайный электрон в неизвестном квантовом состоянии и измеряем его спин по направлению вертикальной оси. С вероятностью 50% мы получим значение спина +1/2 и с вероятностью 50% мы получим значение спина -1/2. Например, мы получили значение спина +1/2. Что происходит в данном эксперименте с точки зрения многомировой интерпретации? Изначально спин электрона находится в суперпозиции, которая может быть задана вектором состояния ( спин +1/2; спин -1/2 ), причём каждое состояние имеет «вес» 50%. После того, как мы внесли в систему измерительный прибор, теперь суперпозиция состоит не только из электрона, а из электрона и прибора. И представляется в виде следующего вектора состояния:

( спин +1/2, прибор показал результат +1/2; спин -1/2, прибор показал результат -1/2 )

Причём каждое состояние опять имеет «вес» 50%

После того, как наблюдатель посмотрел результат измерения, то он перемешался с системой из электрона и прибора и теперь вектор состояния всей системы представляется в виде:

( спин +1/2, прибор показал результат +1/2, наблюдатель фиксирует спин +1/2; спин -1/2, прибор показал результат -1/2, наблюдатель фиксирует спин -1/2 )

И опять каждое состояние опять имеет «вес» 50%

Таким образом, с точки зрения многомировой интерпретации в процессе вышеизложенного измерения вселенная «расщепляется» на две альтернативные ветки, каждая из которых объективно существует и в каждой из которых присутствует свой наблюдатель, наблюдающий электрон в том или ином состоянии. То есть если вы зафиксировали спин электрона в состоянии +1/2, то в альтернативной, но настолько же реальной параллельной вселенной ваш двойник зарегистрирует спин -1/2. Причём с его точки зрения это он будет настоящим вами, а вы — его двойником

Теперь усложним условия эксперимента и ориентируем анализатор под углом 60 градусов от изначальной оси измерения и попробуем провести измерение спина электрона ещё раз. Электрон опять же находится в состоянии суперпозиции, заданной вектором состояния ( спин +1/2; спин -1/2 ). Но теперь первое состояние имеет вес 75%, а второе состояние имеет вес 25%. Далее проводим процедуру измерения, изложенную выше. После чего система представляется вектором состояния:

( спин +1/2, прибор показал результат +1/2; спин -1/2, прибор показал результат -1/2 )

Но в этом случае первое состояние имеет вес 75%, а второе 25%

После того, как наблюдатель снял показания с прибора, то он перемешался с системой и стал находиться в состоянии суперпозиции. Вектор состояния изменился:

( спин +1/2, прибор показал результат +1/2, наблюдатель фиксирует спин +1/2; спин -1/2, прибор показал результат -1/2, наблюдатель фиксирует спин -1/2 )

И опять первое состояние имеет вес 75%, а второе 25%

Рассмотрим, что произошло при данном измерении с точки зрения многомировой интерпретации. С одной стороны по логике многомировой интерпретации вселенная должна была «расщепиться» на две объективно существующих равноправных альтернативных ветви. Первая из которых соответствует первому слагаемому вектора состояния, а вторая — второму. Но одновременно с этим мы должны будем признать, что первая ветка в каком то смысле в три раза более реальна, чем вторая (так как 75% втрое больше 25%). Как это вообще понимать? Если многомировая интерпретация верна, то в каждой из ветвей будет находиться свой наблюдатель, который на полных правах считает именно свою вселенную реальной. Из этого противоречия можно попытаться выйти следующим способом — предположим, что при таком эксперименте образуется четыре альтернативных вселенных. В трёх из них наблюдатель регистрирует спин электрона в состоянии +1/2, а в одной из них в состоянии -1/2. Но с точки зрения самой многомировой интерпретации каждому слагаемому вектора состояния соответствует своя параллельная вселенная. Таким образом, ветви должно быть всё-таки две, а не четыре. Но в таком случае нам приёдётся как-то объяснить, что в данном случае означает то, что одна вселенная в три раза реальнее другой. Это противоречие можно усилить, если ориентировать анализатор не под углом 60 градусов, а под углом 89,9 градусов. Тогда одна вселенная при расщеплении будет примерно в тысячу раз «реальнее» другой. Но с точки зрения самой многомировой интерпретации в каждой из вселенных будет свой наблюдатель, для которого всё кажется совершенно нормальным и никакой проблемы нереальности или полуреальности своей вселенной перед ним не стоит
Но в таком случае нам приёдётся как-то объяснить, что в данном случае означает то, что одна вселенная в три раза реальнее другой.
В изначальной формулировке ММИ просто был постулат, что вероятность наблюдателя перейти в одну из своих копий пропорциональна квадрату амплитуды. Статьи про правило Борна выводят его из более простых принципов, но на мой взгляд, проблема до конца не решена. Настоящим решением этой проблемы было бы следующее: существует некий механизм уничтожения вселенных с минимальной амплитудой (например, из-за деструктивного влияния соседних высокоамплитудных вселенных), таким образом, что во всех выживших выполняется правило Борна. В итоге, для 25/75% наблюдателей мир каждого расщепляется еще на 1010..00 вселенных из-за внутренних квантовых процессов, большая часть из них уничтожается, и оставшихся вселенных, соответствующих одному результату измерения, оказывается в три раза больше, чем другому.
>механизм уничтожения вселенных с минимальной амплитудой

Опять ктото скучает по лангольерам)
Опять ктото скучает по лангольерам
Так по-любому будет либо постулат Борна (в упрощенной формулировке, спасибо теоретическим статьям), либо механизм уничтожения вселенных. Вывода правила Борна напрямую из, скажем, КТП я не видел. Механизм уничтожения лучше в том смысле, что он может возникнуть из-за взаимодействия соседних вселенных без привлечения дополнительных постулатов.
Что происходит в данном эксперименте с точки зрения многомировой интерпретации?

В многомировой интерпретации у электрона самого по себе нет состояния (если только вселенная не состоит из одного этого электрона). Есть состояние всей системы (спин, прибор, наблюдатель), какую-то отдельную подсистему извлечь нельзя, она сама по себе не существует. С-но, у нас изначально (на самом деле, еще до того как появился электрон, до того, как кто-то сделал прибор и до того, как родился наблюдатель) есть состояния вида (+-спин, +-прибор, +-наблюдатель).


Рассмотрим, что произошло при данном измерении с точки зрения многомировой интерпретации. С одной стороны по логике многомировой интерпретации вселенная должна была «расщепиться» на две объективно существующих равноправных альтернативных ветви.

В многомировой интерпретации ничего никуда не расщепляется, все про "миры" — это объяснение на пальцах для упрощения, у Эверетта вообще никакие "многие миры", расщепления, копии и т.д. не упоминались, это уже идет "интерпретация интерпретация. ММИ же исходно — это просто формулы как они есть, вот то что написано в формулах квантмеха — так и интерпретируем. Есть набор векторов состояния (все возможные состояния которые вообще могут быть) и каждому из этих состояний в каждый момент времени соответствует некий коэффициент. Все. Больше в ММИ ничего нет.
В определенных случаях мы можем интерпретировать некоторые особенности поведения ВФ как "расщепление", но это уже не относится к самой интерпретации.


Но с точки зрения самой многомировой интерпретации каждому слагаемому вектора состояния соответствует своя параллельная вселенная.

Можно считать что у вас есть "ветвь" для любого возможного состояния. Просто большинство этих ветвей имеют нулевую (либо почти нулевую) амплитуду почти всегда.
Процесс расщепления — это, грубо говоря, когда у вас есть некоторое состояние (которое, к слову, не точечно, в квантовой механике не бывает "точечных" состояний из-за принципа неопределенности) и из этого состояния амплитуда "перетекает" в два каких-то других. При этом ветви до этого существовали и потом (когда система сменит состояние) тоже будут существовать. Просто с низкой (нулевой либо почти нулевой) амплитудой.
Взяли три колбы, налили в одну воды, потом перелили в две других, по половине в каждую. Вот оно "расщепление".

А вот это хорошая публикация про MWI! Спасибо, было интересно.
Однако нас будет интересовать, а почему же новая теория не завоевала умы сразу же (помимо отторжения из-за того, что она “обкуренная”)? Главным образом потому, что она противоречила наблюдаемым фактам.

Это с каких пор? В ММИ вы видим одного единственного кота, без какой-либо декогеренции. Если вы просто спроецируете ВФ на состояние с мертвым котом, то получите, что в этом состоянии у вас ученый видит мертвого кота. Так что ничему ММИ не противоречила.


Таким образом, MWI является абсолютно минималистичной теорией (ее иногда даже называют NULL interpretation).

Это в оригинальной формулировке (без "миров"), а когда мы начинаем интерпретировать определенные классы состояний ВФ как "разделенные миры" (интерпретация интерпретации :)), то тут уже совсем не NULL.

MWI не решает проблему наблюдателя, и с этой точки зрения ни чем не лучше остальных интерпретаций. Кроме того, она приводит к таким интересным эффектам как "квантовое бессмертие", которые полностью выпадают из научного контекста ввиду отсутствия объективности (субъективный идеализм).


Существование этих ветвей неизбежно следует из формул квантовой механики, как структура пространства внутри черной дыры следует из формул Эйнштейна.

Из квантовой механики следует только то, что живомертвого квантового кота наблюдает радостногрустный квантовый наблюдатель. Все остальное — попытки разрубить радостногрустного квантового наблюдателя на двух классических. Только коллапс рубит лишь наблюдателя, а MWI заодно еще и всю Вселенную вместе с ним.

Ветвление веток в MWI различает прошлое и будущее? Если да, то такая интерпретация не нужна. Я не читал первоисточники, но во всех объяснениях которые я видел MWI явно нарушает симметрию между прошлым и будущем без внятных обоснований и это не дает мне с ней согласиться.
Уважаемый автор, Вы просите ответить на вопросы. И, в частности, на следующий вопрос: Мэри — учёный, изучающий цвета, и знает все физические факты о цвете, включая все физические факты о переживании цвета другими людьми, начиная с поведения, который конкретный цвет может вызвать, и вплоть до конкретных последовательностей нейрологических реакций, которые регистрируются при восприятии цвета. Однако она с рождения была заключена в комнате, раскрашенной только в чёрно-белые цвета, и могла наблюдать внешний мир только через чёрно-белый монитор. Когда ей позволят покинуть комнату, то что произойдет? *
Подпись отсутствует
так как она знает о цвете ВСЕ, в том числе, как на цвет реагирует мозг, то она ничего нового не узнает
она обретет новые знания в первый раз, когда увидит цвет своими глазами
Так вот, если мозг Мэри формировался в чёрно-белом мире, то Мэри останется СЛЕПОЙ К ЦВЕТУ. И знай она теорию, или ничего не ведай о цвете — мир для неё всеми красками никогда не расцветёт, так как её мозг формировался в чёрно-белом мире. С уважением, Сергей Фофанов.
НЛО прилетело и опубликовало эту надпись здесь
Потому, что между слоем колбочек и зрительной корой есть ещё много слоёв интеграции и переработки зрительной информации.
НЛО прилетело и опубликовало эту надпись здесь

Ну, формально, это можно измерить, засунув вам в голову маленькие электроды и последовательно действуя на нейроны. Если вы заметили — значит, осознаете, не заметили — не осознаете :)

НЛО прилетело и опубликовало эту надпись здесь
Ну там вопрос некорректный (как и все философские вопросы, ха-ха). Мэри _знает_ всё о цвете и его физиологии. Увидев впервые цвет она испытает новые _ощущения_, какие раньше не испытывала. Являются ли ощущения знанием? Да фиг знает.

Сергей же выше пишет, как я понял, что в отсутствии цвета не разовьются сами нейроструктуры, ответственные за передачу ощущения цвета в мозг. Не думаю, что это так, но во всяком случае это нормальная гипотеза, которая может быть при желании проверена экспериментально.

Чтобы избавиться от этого обстоятельства предлагается (не мной, описанное Вами затруднение известно давно) заменить Мэри на робота с включенным lock на зрении цветов. Тогда и комнату можно не организовывать

Тут я не очень понял. Ведь у робота нет квалии по определению?

Почему? Мы не знаем что такое квалиа, и кто ей может обладать

НЛО прилетело и опубликовало эту надпись здесь
Тогда другой вопрос — почему вы осознаете движение мышц руки, но не движение мышц кишечника? Одна и таже нейросеть.

Зря вы на этот лёд вступили))) Это сполшная физиология. Там чертовски интересно. Но жутко всё сложно. И почти одна сплошная биохимия. Нейросети сбоку-припёку.

Например, осознаёти ли вы, что когда хотите согнуть один палец на руке — вы, на самом деле, отдаёте команду согнуть все пальцы на руке. А где-то там за вас, оставшиеся четыре пальца получат сигналы торможения, которые нейтрализуют сигналы возбуждения к мышцам и сдвинется только один. Поэтому, говорить о том, что мы осознаем, а что нет — можно очень осторожно. Скорее, мы больше задним числом «додумывем», что мы там «осознаём» по уже свержившемуся факту.
НЛО прилетело и опубликовало эту надпись здесь
Где-то есть жесткая граница, чем она обусловлена?

Тут не вопрос в иллюзорности восприятий. Граница ощущений обусловлена физиологией и средой(как внутренней, так и внешней). Концентрацией различных веществ или их отсутствием.
Говорить, о том, что вы вообще хотя бы один нейрон ощущаете — не вполне корректно. Нейроны сами ничего не ощущают. Им приходят сигналы от рецепторов и от других нейронов. Полно всякой разной химии. Результат этого возбуждения — и есть ваши ощущения.

Почему могу сознательно остановить дыхание, но не сердце

Потому, что вы биологически здоровый человек и ваша физиология функционирет нормально. Обратное вам бы вряд ли бы понравилось.

В организме вообще всё влияет на всё. Где начинаетесь «вы, как разумный субъект», а где «продукт эволюции вида как набор белков и сложных биохимических реакций» сложно сказать. Почти всё тело имеет фрактальное строение и границы всего со всем скорей всего такие же фрактальные — т.е. нет чётких границ. Однозначно — мы продукт сложной химии и очень даже себе материальны.

сердце в одной локальной сети с легкими

С чего это?
НЛО прилетело и опубликовало эту надпись здесь
Почему одинаковые на физическом уровне сигналы от глаз и ушей имеют кардинально различные ощущения?

Кто вам сказал, что на физическом уровне сигналы от глаз и ушей одинаковые? Откуда это?

Не всё осознается в организме.

За пределами организма ничего нет. Это легко проверить. Остальное — плод воображения.

Где эта граница ощущений? Вот идет сигнал от глаза, где-то бежит это электрохимический сигнал, допустим от фонарика. Перескакивает с одного нейрона на другой и никто этого не замечает. И вдруг, в какой-то момент, я осознаю это яркое пятно. Почему в одной и том же нейронной сети есть граница?

Откуда вы всё это берёте? Что это за набор слов? С эти даже спорить невозможно…

самое реальное — ощущения.

Принял правильную пилюлю — появились ощущения. Видимо тут же появилась и соответствующая галюцинанциям реальность.

Так понимаю, что «Квалиа» — это такая форма наркомании…

На всякий случай оставлю интересующимся ссылку на курс скучных лекций по теме «Биология поведения человека» www.youtube.com/playlist?list=PL8YZyma552VcePhq86dEkohvoTpWPuauk

Там хоть не так красочно, но всё-таки реалистично объясняются вышеупомянутые вопросы.

Особенно в контексте, этого треда, возможно наиболее актуальной будет вот эта www.youtube.com/watch?v=JW8tTy4mKHk&list=PL8YZyma552VcePhq86dEkohvoTpWPuauk&index=25 Это хоть что-то тогда объясняет.

НЛО прилетело и опубликовало эту надпись здесь
Давайте конкретно, чем они отличаются?

Хотя бы тем, что они идут по разным каналам, попадают в разные участки мозга, и обрабатываются по-разному. Куда уж конкретней про различия.

нейросетка одна, но я осознаю результат активности только ее части, почему?


С чего вы это взяли? Мозг не представляет из себя единый вычислительный центр. Это запутанная сеть сетей. Где-то связанных, а где-то изолированных друг от друга. Расположенных разными слоями. С самыми разными функциями.

Вопрос — почему я ощущаю?

Ответить на этоот вопрос без четкой формулировки тезиса, что такое «Я» и что такое «ОЩУЩАЮ» — никто не сможет. Сколько будет версий, столько и ответов. Кто-то отправит в биологию, кто-то в физику, кто-то рассажет про «торсионные поля» или «квалиа». Тут важен не ответ, а правильный вопрос. В такой форме он лишен смысла.

Вот это всё и есть квалиа.


Фантазии, проще говоря. Почему бы и нет…
НЛО прилетело и опубликовало эту надпись здесь
Мой образ мыслей не биологический, а аппаратный.

Неважно каким образом мыслить — главное чтоб образ мыслей отражал реально протекающие процессы. В этом смысле образов мысле всего два: адекватный объективной реальности и «фентези».

Колебания напряжения идущие от ютуба такие же как и от хабра, на физическом уровне невозможно отличить информацию.

Отбрасывая уровни, вы отбрасываете и значимую информацию — поэтому и не можете ничего отличить. В реальности колебания напряжения содержат много дополнительной значимой информации и проблем с её интерпритацией не возникает ни в электронных ни в биологических системах.

Во-первых, инфа «обрабатываются» во всех нейронах одинаково (не путайте уровни)! И разные участки мозга состоят из одинаковых нейронов.

Оба утверждения неверны. Дальнейший рассуждения на этих «фактах» тоже будут ошибочны.

Во-вторых, вышестоящие уровни не решают вопрос. Любые вышестоящие уровни являются условными (это сложная тема, но это так).

Не очень понятно о каких уровнях вы ведёте речь. Но мозге очень много различных специализированных зон. Не условных, а вполне конкретных на уровне физических органов. И зоны так же оказывают взаимное активное и пассивное влияние друг на друга. И это взаимодействие всего со всем и формирует какую-то итоговую картину.

почему мы одни высокоуровневые сигналы осознаем, а другие нет?

Без формального определения понятия «МЫ» и связанного с ним понятия «ОСОЗНАЁМ» постановка вопроса бессмысленна. Иначе разговор будет о «кислом и белом». Но, поскольку, можно объяснить это в контексте физиологии человека — то аналогия из машиностроения будет такая: По той же самой причине, по которой в исправной машине при нажатии на клаксон издаётся звук, а не загораются фары.

должен быть независимый интерпретатор.

Его существование из ниоткуда не следует. Во всяком случае, в непротиворечивых рассужениях связь до него не просматривается. Эмержентно возникающие свойства у сложных систем объясняют всё гораздо проще. Это хотя бы принципиально и рано или поздно можно узнать.
НЛО прилетело и опубликовало эту надпись здесь
Нет, не содержат ничего магически дополнительного!

Магия не нужна. Самих колебания уже достаточно. Но дополнительной информации там много. Непонятно почему вы считаете, что её нет. По факту она есть. И возможность её интерпритировать любым приёмником обусловлена как раз её наличием. Странно это отрицать.

Чем нейроны в зрительной зоне отличаются от тех, что в слуховой? Давайте конкретно, я уже спрашивал это.


Могу лишь только поториться www.youtube.com/playlist?list=PL8YZyma552VcePhq86dEkohvoTpWPuauk Ну и гугл в помощь.

Например, есть нейрон бабушки. На низком уровне там некий потенциал и некие нейромедиаторы. А на высоком уровне — бабушка.


Да, есть такое заблуждение. Но, к сожалению нейронов на всех «бабушек» и взаимосвязи между ними не хватит. Корректней говорить, что не «есть нейрон бабушки» — а «есть конфигурация нейронной сети», которая при определеённом входе в виде электрохимических сигналов на выходе даст «бабушку». Это сильно не одно и тоже.

Интуитивно все понимают о чем речь. Но никто еще не смог сформулировать ответ. И я просто задаю вопросы, у меня ответа тоже нет.


Чтоб услышать правильные ответы — надо задавать правильные вопросы. Интуиция у всех сильно разная и многие вещи понимаются сильно по-разному. Поэтому без «общего языка», и только на собственной интуиции услышать что-то вразумительное шансы не большие. Как говорил Р. Декарт: «Люди избавились бы от половины своих неприятностей, если бы смогли договориться о значении слов»

Я не про душу, не глупите.

Оговорка по Фрейду?)) Я ни слова не говорил про душу. Что такое «интерпретатор» — вопросов нет. Но интерпретатор чего? Непонятно. Хорошо. Теперь вы пишите — «процесс превращения информации с одной формы в другую — электрохимический сигнал в слово» — ну ок, уже звучит неплохо. Весьма абстрактно и вполне конкретно. Хотя непонятно почему именно только в «слово». Ну да ладно. Но почему у вас именно два и только два вывода да ещё такие — непонятно. У живых существ вот есть нервная система. У кого-то сложнее, у кого-то проще. В целом, неплохо обеспечивает обратную связь с окружающей средой. Вполне себе материальная штука. Очень неплохо справляется с «процессом превращения информации с одной формы в другую». Относительно неплохо изучена и продолжает активно изучатся. Чем не «интерпритатор»? И никакая «панпсихия» не нужна.
НЛО прилетело и опубликовало эту надпись здесь
По вашей логике в одном бите содержится миллиард гигабайт информации. Изучите тему, прежде чем нести бред.

Это исключительно ваши слова, как и о «душе» выше. Перестаньте приписывать людям то, что они не говорили и не могли сказать.

На самом деле приемник содержит всю информацию, а сигнал только активирует ее.

ЗАНАВЕС. Оказывается у меня в телефоне ВЕСЬ глобальный интернет нынешний и будущий и входящий тафик её просто «активизирует»… ё-мое…

В плане информатики это одно и то же, просто разные уровни.

Да уж. Рёбра графа и его вершины это совсем одно и тоже:)

В общем, думаю, что на этой весёлой ноте наш разговор зашёл в тупик. Дальше его продолжать я уже не вижу смысла. Если начать воспринимать всё серьёзно, то случится полная «панпсихия, квалиа и абстрактная потеря информации(что бы это ни значило)» )))

НЛО прилетело и опубликовало эту надпись здесь
А как же
приемник содержит всю информацию, а сигнал только активирует ее


Сигнал точно не содержит никакой информации, нет?)))

У вас в телефоне хранятся программы, буквы, картинки и т.д.


так понимаю, что весь этот диалог (попадает в категорию «т.д.») у меня уже есть на телефоне, включая ваши будущие ответы и утчонения :)

Ведь «известно», что
Напряжение в проводе или колебание электромагнитного поля только активирует отображение той информации, которая уже есть на устройстве .


С вашего позволения я продолжу и дальше «не понимать такую элементарную информатику» ))) Спасибо, что повеселили.
НЛО прилетело и опубликовало эту надпись здесь
любую инфу можно сжать в 1 бит.

Можно-то можно, но зачем? Можно и в 0 :))) Практической пользы в этом никакой. А вот сжать(т.е. преобразовать) без потерь значимой информации(т.е. возможности хотя бы частично вернуться к исходному сообщению, по сути речь о сохранении возможности обратного преобразования) ниже определённого предела вряд ли получится. Впрочем как и строго определить сам нижний предел.

Поэтому, строго говоря, «сжать» как раз и нельзя, а вот «урезать» до 1 бит — запросто, но это уже будет совсем другая «инфа» никак не связанная с иходной.
НЛО прилетело и опубликовало эту надпись здесь
Пример сжатия без потерь.


Это не пример сжатия — это обычная биекция одного 2-х элементного множества на другое ))) в вашем примере нет никакой трансформации информации, поскольку исходное и конечное сообщения, получаемеые при помощи такого преобразования тождественны с точностью «до изоморфизма» :))

Возможно я вам открою тайну, но дело в том, что если между любыми множествами можно выстроить цепочку биективных отображений, то не теряя общности, можно утверждать, что они неразличимы. Соответственно, любое сжатие, которое есть суть сюрьекция одного множества на другое, не может являться биекцией «по-определению» в силу отсутствия сжатия информации как такового.
НЛО прилетело и опубликовало эту надпись здесь
Всякая функция делает биекцию, шах и мат

Отображение множества целых чисел в кольцо классов вычетов по модулю сейчас в печали:)))
НЛО прилетело и опубликовало эту надпись здесь
Как бы категоричное утверждение
Всякая функция делает биекцию, шах и мат
несколько контрастирует с
всякая функция делает соответствие некоторым элементам множества A какой-то элемент множества B, что чаще всего является биекцией .

Но это уже прогресс и я вынужден отдать честь вашей целестремлённости, но пока что мы всё еще так и находимся на этапе, когда "Смешались в кучу кони, люди,..."

Сакраментальный вопрос
кто такой я и что я тут делаю и что такое сознание.
очень хороший и очень нетривиальный. Вы большой молодец, что активное иго исследуете. Но в выборе методов исследования и базовых предпосылок, были бы вы поосторожней. Боюсь, что кроме инструментария старого доброго, скучного и занудного "научного мышления" выбрать и нечего. Все остальные пути к ответам хоть и сильно короче, но столь же тупиковы(если, конечно, вам интересен настоящий ответ а не его химера) И, вдобавок, в отличие от других, никто даже не гарантирует, что вообще ответ будет найден. Но сам поиск может быть вполне увлекателен.
НЛО прилетело и опубликовало эту надпись здесь
НЛО прилетело и опубликовало эту надпись здесь
Пожалуй, я всё-таки останусь при своём мнении:) Спасибо.
Потому что сигнал от глаз — это кабельная высокочастотная антенна, а сигнал от ушей — телефонная лапша.

Кстати, слепым делали прибор дающий в уши Ч/Б развертку навроде SSTV, они прозревали (те у кого ранее было зрение и полноценно развился соответствующий участок коры). В массы не пошло, так как слепые предпочитали не засорять привычный канал.
В основном — обратные связи. Дай человеку возможность напрямую наблюдать сердцебиение — он сможет его произвольно регулировать.

Во вторую очередь — автономные программы (рефлексы). К примеру, рефлекторное закрывание глаза нельзя прекратить силой воли, и тренировки не помогут.
НЛО прилетело и опубликовало эту надпись здесь
Интересные вопросы. Вряд ли на «самый главный вопрос» пока что есть ответ.

Но кое что из известной физиологии можно предположить:
Скорей всего нет чёткой границы между частями мозга «вот тут человек», а вот «тут не человек». Как нет чёткой границы «появления сознания» между «плодом» и разумным «сформированным человеком» — никому не известен факт, что вот секунду назад был бессознательный клубок белков, а вот тут уже что-то стало что-то «разумное».
Вот возьмите, например, фрактал в виде Множества Мандельброта — известно, что за кругом радиуса больше 2 его нет, а около точки (0,0) он точно есть — но где между 0 и 2 он «начинается» зависит от масштаба и других деталей, которые принимаются во внимание.

Про «возможность наблюдения», в контексте вышесказанного вообще сложно тогда говорить — непонятно что имеется ввиду.
Это вопрос терминологии, корректнее в моем посте «человек» заменить на «осознанно». Неосознанно может конечно, и регулирует — и сердцебиение и дыхание, все мозгом регулируется.

Сознание это прогностическая модель, в рефлексах нет прогнозов, это простая программа if else. И таки да, человек это набор таких вот скриптов, «написанных» как на языках нервных импульсов, так и на биохимических принципах.
Это точно ко мне вопрос? Я не нейрофизиолог, не знаю. Но в отличие от квалии, на этот вопрос, опять же, можно ответить научными методами.
Спорный вопрос. Вот сидят в бункере два человека, один физик-ядерщик и биолог, а другой обыватель. В некий момент они выходят в радиоактивный мир и один видит радиацию (как вспышки в голове), а другой просто видит вспышки, но не догадывается о радиации.
Идея физиков проста: вам шашечки или ехать? Меня в МИФИ учили нахождению физ-смысла в математических выкладках. Увы в квантовой механике, похоже, физики вынуждены были от этого отказаться. Надо ехать — берите формулы и поезжайте! Счастливого пути и скатертью дорога.
Tzimie
Или всю жизнь крутиться в химерах типа коллапса, спутанности, мгновенно менящей состояние спутанных частиц, как бы далеко они ни были, или корпускулярно-волнового дуализма.


Вспомнилось мне как я год назад писал о том, как физики избавились (или попытались избавиться) от корпускулярно-волнового дуализма.

История эта давняя и была прочитана мною в «книжке для любознательных» изданной ещё в СССР.

Реально (обычно на экране за мишенью) мы видим либо интерференцию либо её отсутствие. — Логично первое считать проявлением волны, а второе неделимой частицы. Так и порешили физики вначале. Но потом…

Возьмём стандартный, из учебников, пример — прохождение квантовой частицы через две щели. Если «смотрим» (датчиком) через какую щель прошла квантовая частица — то квантовая частица проявляет свойство неделимой частицы, если «не смотрим» (датчик убираем) через какую щель прошла квантовая частица — то свойство волны (на экране после прохождения щелей виден рисунок интерференции).
Казалось бы корпускулярно-волновой дуализм на лицо! Но давайте немного подумаем и… усложним опыт:

Мы многократно увеличим число «щелей» и уберём от них наши датчики (датчик — это некий прибор, передающим нам (экспериментаторам) сигнал о прохождении квантовой частицы через щель).

Для этого мы будем рассеивать пучок очень медленных (энергия порядка 0,1эв) нейтронов на кристалле. — Это реальный опыт, а не выдуманный. — Роль щелей будут играть… ядра атомов кристаллической решётки кристалла. Нейтрон может рассеяться на ядре двумя способами — с переворотом спина или без переворота спина. (К примеру, летят два нейтрона, у одного спин вверх, у другого вниз, оба столкнулись с ядром кристалла (произошло рассеяние на ядре кристалла), у одного нейтрона спин перевернулся от этого — и дальше полетели оба нейтрона с одинаковым направлением спина и прямо в экран и «впечатались»). Важно тут понять понятие «различимая альтернатива» и «неразличимая альтернатива».

«Различимая альтернатива» наступает тогда, когда после рассеяния у нейтрона и ядра атома кристалла спины поменяли направление (ну, хоть как-то).
«Неразличимая альтернатива» наступает тогда, когда после рассеяния у нейтрона и ядра атома кристалла спины никак не изменились (вообще), хотя возможно изменилась траектория нейтрона, летящего к экрану.

Вы, надеюсь, узнаёте в поведении ядра атома кристалла тот самый «датчик» из классического опыта с двумя щелями. — Только в нашем опыте ядро атома кристалла никак не связано с чем-то, что подаёт сигнал экспериментатору. Но несмотря на это «датчик» этот «работает».

Проведя опыт мы увидим на мишени некую интерференционную картину.

Как её объяснить?
— Без привлечения корпускулярно-волновой дуализма можно так:
Если при рассеянии спин нейтрона переворачивается, то «альтернативы различимы» и мы складываем вероятности волновой функции. (сумма квадратов модулей волновой функции) — (эти нейтроны порождают фон на экране)
Если при рассеянии спин нейтрона НЕ переворачивается, то «альтернативы НЕразличимы» и мы складываем амплитуды вероятностей волновой функции. (квадрат модуля суммы волновых функций) — (эти нейтроны порождают интерференционные полосы на экране).

И всё — никакого корпускулярно-волновой дуализма тут не используется при описании этого реального опыта.

Попробуем объяснить картинку с привлечением корпускулярно-волновой дуализма, например так:
Если при рассеянии спин нейтрона переворачивается, то нейтрон проявляет свойства частицы — (эти нейтроны порождают фон на экране)
Если при рассеянии спин нейтрона НЕ переворачивается, то нейтрон проявляет свойства волны — (эти нейтроны порождают интерференционные полосы на экране).

Что использовать — выбирать вам. Но имеет ли смысл (кроме убеждений глубинного вашего философского плана) считать тот или иной нейтрон волной или частицей только потому что у него при рассеянии перевернулся (или нет) спин? — При этом неважно само направление спина нейтрона «впечатывающегося» в экран (для картинки на экране) — а важен сам факт случившегося некого квантового события — переворачивался ли спин при рассеянии или нет.

Когда физикам «надоело» всюду писать что что-то «проявляет свойства частицы» или «проявляет свойства волны», они поняли что можно просто писать «складываем вероятности волновой функции» или «складываем амплитуды вероятностей волновой функции» — таким образом произошло избавление от корпускулярно-волнового дуализма.

Потом физики успешно переименовали "Волновую механику" в "Квантовую механику".
А вот переименовать "волновую функцию" в "«амплитуду плотности вероятности»" не смогли — мем твёрдо засел в мозгах (вспоминая Докинза).
А вот переименовать "волновую функцию" в "«амплитуду плотности вероятности»" не смогли

Ее переименовать нельзя, потому что волновая ф-я амплитудой не исчерпывается, у нее еще фаза есть.


можно просто писать «складываем вероятности волновой функции» или «складываем амплитуды вероятностей волновой функции»

Вы бы уточнили, что под чем подразумеваете.

Ее переименовать нельзя, потому что волновая ф-я амплитудой не исчерпывается, у нее еще фаза есть.


Ошибка моя. Правильно читать: А вот переименовать "волновую функцию" в «амплитуду вероятности» не смогли…

Так вернее будет то. Хотя «амплитуда вероятности» также встречается в статьях, вместо "волновая функция", но реже, гораздо реже.

Вы бы уточнили, что под чем подразумеваете.


Ошибка моя. Правильно читать:
«Когда физикам «надоело» всюду писать что что-то «проявляет свойства частицы» или «проявляет свойства волны», они поняли что можно просто писать «складываем вероятности» или «складываем амплитуды вероятностей» — таким образом произошло избавление от корпускулярно-волнового дуализма.»

Вообще когда летят два нейтрона с одним и тем же спином (величиной и направлением) и один говорит другому:
я волна, ибо я рассеялся без переворота спина.
А второй ему отвечает:
я частица, ибо я рассеялся с переворотом спина.

И тут невольно задумываешься, а чем же они отличаются друг от друга кроме как… "историей своей"?

P.S. Нашёл на просторах сети:
"… после открытия уравнения Шредингера самым сложным было понять физический смысл волновой функции. Сам Шредингер вначале неверно истолковывал смысл волновой функции.

Волновая функция описывает не обычные материальные волны, как электромагнитные или волны на поверхности моря, а волны особые — «волны вероятности». Они описывают амплитуду вероятности распределения частиц в пространстве.

Связь между функцией распределения вероятности и волновой функцией установил в 1926 году М. Борн. Это один из самых глубоких и загадочных принципов в квантовой физике — правило Борна (в честь Макса Борна). Правило очень простое — вероятность получения любого возможного результата измерения равна квадрату соответствующей амплитуды вероятности (квадрату модуля волновой функции). Это правило не выводится, не является теоремой и не имеет теоретического обоснования, но дает практически математически строгое совпадение результатов экспериментов с ожидаемыми значениями, рассчитанными в рамках уравнения Шредингера.

Примечательна история публикации правила Борна. Сначала статья его была отклонена журналом. Затем была принята другим журналом, но в статье было напечатано, что вероятность равна амплитуде (это была очевидная опечатка, так как амплитуда может быть отрицательной и даже мнимой), и только позже, в добавленной автором сноске, он исправил её как квадрат амплитуды.

Такие вот дела с квадратом волновой функции."

Публикации